Sunday, September 5, 2021

Make-believe matter

Materialism can at first blush seem to have a more commonsensical and empirical character than Cartesian dualism.  The latter asks you to believe in a res cogitans that is unobservable in principle.  The former – so it might appear – merely asks you to confine your belief to what you already know from everyday experience.  You pick up an apple and bite into it.  Its vibrant color, sweet taste and odor, feel of solidity, and the crunch it makes all make it seem as real as anything could be.  Anyone who says that all that exists are things like that might, whether or not you agree with him, at least seem to have the evidence of the senses in his corner. 

The trouble is that that is not what the materialist is saying.  The matter to which he would reduce everything is not the matter of common sense, not the hard earth of daily experience.  It is instead a highly abstract theoretical construct which – just like Descartes’ res cogitans – is not and indeed cannot be known directly via perception (nor, unlike the res cogitans, by introspection either).  Moreover, it is a conception the materialist has inherited from Cartesian dualism itself.  And it is that conception of matter, rather than the Cartesian’s commitment to a non-empirical res cogitans, that has made it so difficult for Cartesians and materialists alike to account for how conscious awareness relates to the physical world.

Longtime readers of this blog will recognize these themes.  They are also among the themes of philosopher William Barrett’s interesting 1986 book Death of the Soul: From Descartes to the Computer.  Barrett writes:

Oddly enough, the trouble with the Cartesian dualism comes from the side of the body.  The body, as Descartes conceives it, is not such that it can accommodate the soul.  It cannot, so to speak, be penetrated by the soul; it can only remain in external contact with it.  This body is not the physical body, our physical body, as we know it in our daily intimacy with it.  It is the body of physics – that is, of the science of physics; a piece of matter, and particularly as Descartes conceived of matter.  But the body of physics is remote and unknown to us and is not the body we live in in our day-to-day existence.  The body we know is rarely sharply distinguishable from the soul: in our moods and feelings we are not often sure what part is physical and what not.  There is no sharp dividing line between.  The life of flesh and blood is particularly focused about the feelings and emotions.  So long as there is no adequate conception of the concrete or lived body, our theories of mind cannot deal adequately with the life of feeling. (pp. 19-20)

Let’s pause over the second half of this passage.  Consider the feelings and emotions people experience every day – the warmth in the throat that results from a swig of hot coffee, a twinge of pain in the knee or lower back, an itch from a rash that won’t go away, a pinch from tight clothing, the twinge of excitement that spikes in the chest upon being startled, the tightness and heaviness in the chest that can result from worry or depression, the uncomfortable fullness that results from having overeaten, the pressure of having too full a bladder or colon, the agitation of sexual arousal, anxiety, fear, or anger, and so on.  All of this is experienced as bodily.  It has the feel of knowing a particular bit of matter from the inside, as it were, from a place deep within the marrow and viscera.  If you were unacquainted with the way modern philosophers talk and one of them told you that these features of daily life were “mental” or “in the mind,” you would suppose that what he meant is that they were somehow unreal or imaginary, which sounds crazy.

But that they are in the mind rather than in the body is precisely what post-Cartesian philosophers, whether dualist or materialist, do say.  They take matter to be devoid of anything but primary qualities such as size, position in space, motion, and in general what can be given a purely mathematical characterization.  Anything else is treated as merely a projection of consciousness.  Hence the redness, sweetness, fragrance, and crunchy sound of the apple are taken to be nothing more than the qualia of our experience of the apple, and not to correspond to anything in the apple itself.  And in the same way, warmth, pain, itches, twinges, sensations of pressure, tightness, fullness, arousal, and the like are taken to exist only as the qualia of our conscious experience of the body, and to reflect nothing in the intrinsic nature of the body itself.  The matter of the apple and that of the body alike are taken to comprise nothing more than colorless, odorless, tasteless particles moving through space.  The purely mathematical description of matter afforded by particle physics exhausts their nature. 

Hence, on this view, introspection no more reveals to you the nature of the flesh than perception reveals to you the nature of the apple.  Apple and flesh alike are strictly unobservable.  What we perceive and introspect are really just the mind’s representations of the apple or the flesh, and not the real McCoy.  Physical theory, as interpreted through a mechanistic philosophy of nature, tells us their real nature. 

Now, Berkeley had a field day with this picture of matter and our knowledge of it.  Though he famously denied the reality of matter, the matter he denied was already a pretty ghostly thing anyway – matter stripped of all the concrete reality perception and common sense attribute to it, leaving only a desiccated mathematical husk.  And Berkeley’s point was in part that there is nothing left for the mathematical structure described by physics to be the structure of when all of those concrete features are abstracted away. 

We are accustomed to treating Berkeley’s philosophy as slightly (or more than slightly) mad, when the truth is that he was merely drawing out the implications of the mad conception of nature bequeathed to him by Descartes, Locke, and company.  As Barrett writes:

The real world of our common experience contains trees, grass, singing birds; houses and other people; chairs and tables; etc., etc.  In our daily life these are evidently and substantially there, and not at all “subjective” appearances of something else; and for Berkeley, too, they are evidently and substantially there.  It is Locke who would undermine their reality, and make it secondary to some underlying abstractions of physics.  And here Berkeley turns Locke’s own empiricist weapons back upon their originator.  (p. 39)

Specifically, Locke and company had appealed to the way that the appearances of color, sound, and the like are relative to the perceiver as a reason to judge them mere secondary rather than primary qualities of matter.  Our ideas of motion, position in space, etc. correspond to something really out there in mind-independent reality, but our ideas of color, sound, etc. do not.  Berkeley argued that the appeal to relativity can be deployed against all the so-called primary qualities as well.  And that includes what the Newtonians naively took to remain objectively out there after the mechanists were done with their desiccation of matter.  Here Barrett is worth quoting at length:

Here Berkeley introduces the principle of relativity in a bold and thoroughgoing form that was not to emerge again until Einstein in the twentieth century.

The motion or rest of material particles had been taken as absolute by Locke because there was the absolute space of Newton in which they moved or remained at rest.  And here Berkeley performs one of his most audacious acts of analysis, as he seeks to pull down one of the sacred pillars of the Newtonian world.  Space – the absolute space of Newton, container of all that is – is not given as a reality in and of itself.  It is built up as a high-level abstraction from our perceptions of touch and vision.  It is derived from experience; it is not the container of experience.  We invert the proper order by taking the abstraction as a concrete reality.

In sum, the whole world of matter, which Locke would make the substratum, or underlying reality, for the world of our common experience, is in fact a high-level intellectual construction.  It is a case of misplaced concreteness, as the philosopher A. N. Whitehead in our century has called it: the abstract concepts of physics are taken as ultimately concrete in place of the ordinary world of common experience.  Berkeley stands with this ordinary world, and he consistently reassures his ordinary reader that he is on his side against the materialism of sophisticated philosophers.  (p. 40)

The trouble, of course, is that Berkeley himself is not entirely the friend of common sense, to say the least.  For he denies that tables, chairs, and the like continue to be there when no one perceives them, and that thesis is no part of common sense.  Like common sense, Berkeley holds that there is no real distinction between primary and secondary qualities.  But whereas common sense takes all of these qualities to be out there independent of the mind, Berkeley takes them all to be equally mind-dependent.  Like his philosophical predecessors, he buys into the basic move of the moderns, which is to relocate the qualities of physical objects into the mind.  It’s just that, unlike those predecessors, he relocates all of them there.  Hence even Berkeley is in fact insufficiently radical in his critique of his predecessors, precisely because he is insufficiently reactionary. 

Barrett makes the important point (at pp. 40-41) that the contemporary materialist’s obsession with the computer model of the mind in no way remedies the tendency of the modern conception of matter to collapse matter into mind.  For the notion of a computer is itself the notion of something essentially mind-dependent.  Nothing counts as hardware or software apart from an intelligence which has designed them to function as such.  (This is a theme that John Searle would go on to develop in detail a few years later, in argumentation that I have discussed and defended at length elsewhere.)

When I say that the moderns either partially or wholly move the qualities of physical objects into the mind, what I mean, to be more precise, is that they move them into the intellect (albeit the empiricists on the list, unlike Descartes, collapse the intellect and the imagination, but put that aside for present purposes).  That is why Descartes notoriously takes non-human animals to be insensate automata.  They lack rationality, hence they lack a res cogitans.  Thus, since for Descartes the only other kind of substance there is is res extensa, which is pure extension devoid of any consciousness, that is what animals must be. 

This too is contrary to the commonsense conception of matter.  Consider again what you experience in your own body, and experience precisely as bodily phenomena – sensations of warmth, flashes of pain, twinges of excitement, arousal, fear, and anger, and so on.  Common sense takes something like all this to exist in dogs, horses, bears, and other non-human animals, even though these creatures lack rationality and are entirely corporeal in nature.  For again, from the point of view of common sense, these features are bodily in nature, not mere representations in the intellect, so that other bodily but non-intellectual creatures are as capable of exhibiting them as we are.

Some contemporary philosophers, cognizant of the problems with the early modern mechanistic and mathematicized conception of matter, have reinserted into matter the qualities common sense attributes to it, but then fallaciously draw the conclusion that this entails panpsychism.  They are making a mistake similar to Berkeley’s, and like him they are insufficiently radical precisely because they are insufficiently reactionary, accepting as they do too much of the modern conception of matter they claim to be rejecting.  For like the early moderns, they take the qualities of ordinary physical objects to be partially or wholly mind-dependent, i.e. to be identified with the qualia of conscious experience.  Unlike the early moderns, they take these qualities to exist in physical objects themselves, and not just in our minds.  The result is that they conclude, absurdly, that there must be something analogous to conscious awareness even in rocks, dirt, tables, chairs, etc.  (The poor moderns.  They just can’t do anything right!)

Another part of the problem is that they implicitly buy into the modern reductionist idea that all matter is essentially of the same one type.  Hence if there really is consciousness in the matter that makes up non-human animal bodies, then (the fallacious implicit inference goes) there must be something like it too in rocks, dirt, tables, chairs, etc.

The sober, boring truth – enshrined in Aristotelian philosophy and common sense alike – is that some kinds of purely material substances (namely non-human animals) are conscious, and others (like rocks and dirt) are not.  The latter really do possess qualities like color as common sense conceives of it, but that does not entail panpsychism, because (contra Descartes, Berkeley, and company) those qualities are not entirely mind-dependent.  Not all matter is reducible to one, lowest-common-denominator type, and none of it is reducible to the purely mathematical description afforded by physics.  That description is merely an abstraction from concrete physical reality.  It captures part of that reality, to be sure, but not the whole of it. 

To think otherwise is somewhat like thinking that “the average person” of the statistician really exists, but that the various individual people we meet from day to day do not.  The reality is that those individuals do exist, and that the notion of “the average person,” while it captures important aspects of reality and is therefore useful for certain purposes, is a mere abstraction that does not correspond to any concrete entity.  And in the same way, the concrete physical objects of everyday experience also really do exist, whereas the mathematical description afforded by physics, despite its undeniable predictive and technological utility, does not capture the entirety of concrete reality.  To pretend otherwise is, we can agree with Berkeley, a kind of make-believe. 

Related posts:

Metaphysical taxidermy

Aristotle’s Revenge and naïve color realism

The particle collection that fancied itself a physicist

Materialism subverts itself

Concretizing the abstract

Chomsky on the mind-body problem

Nagel and his critics, Part VIII

Schrödinger, Democritus, and the paradox of materialism

181 comments:

  1. At first glance, materialism seems to be wiser and more effective than the two Cartesian offshoots of mechanism and idealism. It even implies belief in the goal of self-disclosure, in contrast to the hermetic sealing away of mind and world. However, some difficulties exist.

    You can limit your knowledge of your daily experiences in advance; you can cut apples which, aside from their color, delicious taste and aroma, have a strong emotional structure integral with your body. Whether you agree with it or not, people can find such emotional evidence (evidence of a non-material reality under materialism) in the corner of their experience

    ReplyDelete
  2. I've always wondered what the relationship is between between material realities like color (a quality) & spatial extension and immaterial things like form and others. Because spatial extension & color don't seem like they are potencies or purely limitations of form - they strike us intuitively as positive realities with their own content, definition and even form (spatial extension belongs to the category of quantity, and if it's also a necessary part of material essences, it clearly has a form of its own since non-actuality can't be a part of the immaterial essence of a thing). A body for example isn't just a particular limitation of the universal human into a specific location, but its physicality is also a real thing

    If that's the case, why are spatial extension & color uniquely "material" in contrast to the immaterial? What about them, if they are actualities in their own right and their own category, makes them such that God doesn't have them since He is immaterial? And if God is pure actuality, why does He so to speak "lack" spatial extension and color, since all actualities pre-exist eminently in Him in some way, and He has all actualities?

    ReplyDelete
    Replies
    1. Extension do seems a limitation-only thing. I mean, a body just is something whose range of action, contra something like a angel, is perpetually "stuck" on a certain level. The fact that it is a quantitative feature also says that, for something that is of a certain quantity is also "stuck" on a particular level.

      Color seems like a type of way of perceiving this limit of extension on us and others or a way of the body itself to "signal" this. It is not something that a more "free" being like a angel can have, so it seems plausibly a thing only to limited beings too.

      Maybe these things look substancial for us because they are our primary object of perception. The angels, who start with universals, probably see the material as pretty puny.

      Delete
    2. @Talmid,

      Extension may have limiting properties, but is it really PURELY limiting, and just solely potential? Because space and geometrical dimensions are a real things - they even have definitions, which seems to imply they have their own forms. Quantity in general seems even more likely to have positive reality, insofar as even Aristotle puts it as a category of being.

      The finite may not be infinite, but it's still real - or at least, whatever it is that fills the finite "space" isn't a lack.

      As for color - well since qualia aren't purely mental but are really out there in the physical world itself, they are real properties of material things like wetness or mass - at least, it's a property of certain wavelengths of light. I don't get what you mean by saying that it's a "type of way of perceiving this limit of extension on us".

      In fact, if you think about it, red light and blue light all have their own natures, and so they have forms, which means they aren't just purely potential. What do you think?

      Delete
    3. JoeD: "Because spatial extension & color don't seem like they are potencies or purely limitations of form [you seem confused; a potency is not a limitation of form; an actual limitation of a form is something itself formal, itself a form, on the side of act, not potency] - they strike us intuitively as positive realities with their own content, definition and even form."

      Right, they are accidental forms of material substances. They are on the side of actuality, not potency. They are "purely material" only in the sense that they are not separable in their own existence from matter.

      Delete
    4. @David,

      That makes sense. In that case then, why exactly would God as pure actuality lack those accidental forms? Does His containing them in a virtual or eminent way fulfill that already? Or is there something about greater types of actuality that are opposed to accidental forms of matter?

      Delete
    5. "Because space and geometrical dimensions are a real things - they even have definitions, which seems to imply they have their own forms."

      As said by David, these things are acidental forms. Since in God there can be no acident, them God can not in principle have these things. Angels also can't have these because to have a spacial dimension you need to be "stuck" on a certain place, which they are not. It is a bit like having wings: a real feature yes, but of lower beings.

      From the fact that these things have a definition it does not follow that they are perfections(which is what God needs to have) for they limit the being that have they. For instance, animality has a definition but God does not have it because having animality is being limited on a certain way. Hell, onr can argue that having a definition is only really possible to limited beings, but that is another issue.

      About color, yea, i did not spoke very well there. I guess that one could say that color is a way of "sinalizing" or marking the being limits. It is hard to describe qualia, things like mary room or even trying to communicate a sensation you have
      (Like a certain taste) to someone who never experienced something similar shows that our language is very bad at descriving sensations.

      Could this be, like something you hear from a neoplatonist, because it is on a very base level of being and so of integibility? Now i thought in that and it is a interesting question.

      Delete
  3. @JoeD,

    God "lacks" spatial extension as an infinite unbiased random sequence "lacks" one of its finite disordered subsequences. It doesn't.

    Color is a psychological effect of the mind, but not in the modern, self-contradictory psychology of today, but in the true, pre-modern, Aristotelian-Thomistic scolastic psychology.

    Tom Cohoe

    ReplyDelete
    Replies
    1. Color is a psychological effect of the mind,

      Would it be better to say "color HAS a psychological effect on the mind..." I think the A-T position is that color exists in things like the ball or the snow, even if the manner in which it exists IN colored things is distinct from the manner in which it exists in the mind.

      Delete
  4. It is seeing how bizarre Descartes metaphysics are that i few like treading, say, platonic or hindu dualism like versions of cartesian dualism is quite a disservice to these earlier thinkers. Are there ways on the literature to separate these types of dualism?

    Anyway, good post, especially the Berkeley part. Dude sure got pretty far with the paradigm he had, i even remember once reading a note of his were he seemed to catch the contradiction(latter mentioned by Hume) between empiricism and a realist view of personal identity(the "I"). A smart guy he was, thought not enought to just read the classical and medievals while paying attention.

    Modern philosophy seems to be full of cases where someone has a pretty dumb idea but never see the implications, so latter thinkers see and go along with it. It tends to generate funny stuff but sadly these errors are sometimes believed by people.

    ReplyDelete
    Replies
    1. @Talmid,

      "Are there ways on the literature to separate these types of dualism?"

      If you want to think about and distinguish the ways of being wrong there will be no end to it, because there is no end to the ways of being wrong - but there is only one way to be right. So the literature will be huge, containing all the errors significant in cultural impact as well as insignificant - and then it must also contain all the demonstrations that make manifest the error as well, important work in damning up the ramifications of each error into the minds of the nearly innocent.

      By "significant in cultural impact" I mean up to and including errors that distinguish major religions such as Hinduism from Greek paganism. "Insignificant" goes all the way down to two guys disagreeing in front of an audience.

      Well, happy distinguishing so long as you do not lose the Way! :-)

      Tom Cohoe

      Delete
    2. The Way is what we need to truly look for, yes. But my boy Plato sure is screwed by being put on the same team as Descartes when his position is diferent.

      How could i not want to correct this mistake when the wrestler gave us do many cool dialogues? It seems nicer to give his position a diferent name.

      Delete
    3. @ Talmid,

      Ha - ha. Yes Plato and Descartes belong under the same name "dualist" as "soup" includes fetid miasma and something good to eat. They just ain't the same thing but this language confused world is the one we have to work in. :-)

      Tom Cohoe

      Delete
  5. For Descartes, of course, the uncommonsensical nature of matter was a feature, not a bug: by denying all but "clearly and definitely conceived," mathematicizable, properties to matter he was clearing the way for a deductive and quantitative physics. One of the benefits, for him, of rehearsing the sceptical commonplaces of the First Meditation was that it made it easier for the mind to detach itself from (take leave of?) the senses. After which it could do real physics and not just phenomenology.

    ReplyDelete
  6. What's very interesting in contemporary philosophy is the proposition that consciousness and lived experience are synonymous in content and meaning, gaining increasing interest particularly in the cognitive sciences. It is called the Integrated information Theory (IIT), connecting consciousness to physical causality. The proposition is that the 'hard problem' of consciousness may not be one that is at all 'hard'. Rather, the bifurcation of the 'mind-body' problem is more likely the wrong way of looking at it.

    THIS ARTICLE outlines, in part, Professor Giulio Tononi’s Integrated Information Theory, explaining the phenomenon of consciousness. As noted, Tononi's proposition is gaining rather significant support as an explanatory model of consciousness.

    The abstract from THIS SECOND ARTICLE notes:
    "Integrated information theory (IIT) has established itself as one of the leading theories for the study of consciousness."

    The theory is still in its infancy but its formative developmental stage is indeed encouraging towards understanding what consciousness is.

    Happy reading.

    ReplyDelete
    Replies
    1. @Papalinton

      "the cognitive sciences"

      Always a swamp of breathlessly proclaimed "disciplines" that are not. Always has to be faced and watched and answered to prevent "1+1=3" from growing into a fetid swamp sucking at almost innocent minds.

      Let me be clear, I am not in any way opposed to the science that traces the flow of sensory input through various neural pathways through the brain to various destinations in the brain. But that this science studies "cognition" is a misleading delusion. Bouncing information around is "cognition" only to those who beg the question.

      Tom Cohoe

      Delete
    2. BalancedTryteOperatorSeptember 7, 2021 at 12:40 AM

      Cognition is self-awareness. That's why the test for cognition in animals is whether they can recognize themselves in the mirror. It is the point where your mind fills your body up to the tip of your fingers and toes.

      Delete
    3. Papalinton,

      Hmmm, researching IIT I find that the theory has panpsychist implications which (in the first article you link to) "Tononi and Koch unabashedly embrace." Searle says the same thing.

      Others say that the axioms of the theory aren't really axioms. Neuroscientist Michael Graziano says IIT is "pseudoscience" and a "magicalist theory". Given the panpsychist implications above Richard Carrier would define it as "supernatural". How interesting.

      Delete
    4. Agreed DrYogami. At the very least, it is metaphysics/natural theology masquerading as neuroscience. They should firmly develop the first before delving into its implications on the later.

      Delete
    5. "Cognition is self-awareness"

      As a person is a toenail, complete with its thickness as a binary measure of existence. Such nonsense you spew.

      Tom Cohoe

      Delete
    6. @ Anonymous at 12.18AM

      I am somewhat disappointed in the way you have expressed your deep antipathy to the sciences and your profound lack of understanding how scientific research is carried out. Here we have the emergence of a robust explanatory model that has brought together most, if not all, not only the known necessary elements of what constitutes consciousness, but also addresses those once deemed seemingly disparate elements, such as the relationship of qualia and physical causality, into the theory.

      IIT has taken the science of consciousness to another level of understanding, from both the lab, and from the semantical philosophy of Aristotle, Aquinas and Cartesian dualism, into the language of mathematics.

      Of course, there are many detractors and (bad faith?) distractors of all kinds who think they have taken down the IIT. But it is another careful step to a deeper understanding of what constitutes consciousness. And just as Einstein's theory of relativity conjoined two (at the time) very disparate and challenging concepts, energy and mass as synonymous, in a very simple and yet profoundly exquisite and sophisticated formula, E=mc^2, so too are we on the way for a theory of consciousness be described, mathematically. And it won't be anything like Aquinas's medieval nonsense of being 'God endowed'.

      Delete
    7. @ Papalinton,

      Simplification is the key concept below.

      I signed myself "Tom Cohoe", not "Anonymous at 12.18AM" so why not address me as "Tom Cohoe"? Then, the conversation could be followed easily by anyone interested.

      "I am somewhat disappointed [...]".

      You must bear a heavy burden of disappointment if everyone who does not follow your favored construction is projected in your mind as being "antipathetical to science". However that I "profoundly lack an understanding of how scientific research is carried out" is just a hostile personal comment since you know nothing about what I understand of scientific research. You'll look better if you keep _ad hominems_ to a minimum.

      "E=mc^2" has nothing to do with this. Citing it does not convert your enthusiasm into real science. However, since you bring it up, it is better expressed as "E=m" as c is just a constant whose numerical value depends on the space-time co-ordinate frame chosen (see "Spacetime Physics" by Edwin Taylor and John Archibald Wheeler). "E=m" is a co-ordinate independent expression in special relativity. E is then just the projection of the 4-vector, momenergy (again, see Taylor and Wheeler), onto the local time part of the Minkowski frame and mass is the projection onto the space part.

      Not only is this a great simplification in special relativity, it expresses the mathematical equivalence of mass and energy in a locally flat manifold as clearly as it can be mathematically expressed.

      You do understand that the way scientific research is carried out is as a method of simplifying complex raw data into simple expressions that the human mind can comprehend, do you not? The human mind is not very good at understanding complexity, except perhaps in your case - (sorry that was sarcasm, but, you know, you made up from the blue that I know nothing of scientific research so how could you expect to be answered).

      "Aquinas's medieval nonsense of being 'God endowed'".

      Maybe you should try reading Aquinas instead of merely mocking him. 'God' is the greatest unification, in our understanding, of reality as we can perceive it and think about it. Unification (read simplification) is also the goal of science. Nothing Aquinas ever said or did was against science, but he had a remarkable ability to incorporate the science of the day into his finite understanding (that's all any of us has) of the ultimate simplicity, God, because Aquinas' ability to talk about limitless infinity is great. He would incorporate the science of today just as well were he around now (I hope you do not think that real science did not exist before Aquinas).

      If you read Aquinas for understanding you will see expressed in a verbal way, ideas that mathematicians were unable to express for centuries, such as the ideas of open sets, closed sets, limit points in sets that do or do not contain their boundaries, etc. He adumbrated Newton's laws of motion.

      Don't escape thinking through use of scorn and mockery. Embrace it.

      Tom Cohoe

      Delete
    8. @ Papalinton,

      I did not sign myself "Anonymous at 12.18AM" but as "Tom Cohoe". Why not address me as such? Following the conversation would become much simpler.

      "I am somewhat disappointed [...]"

      You must bear a heavy burden of disappointment if everyone who does not follow your favored construction must be projected as "antipathetical to science". That I "profoundly lack understanding of how scientific research is carried out" is something you cannot know. It is mere hostility to me. You would look better if you kept _ad hominems_ to the minimum level that your emotions can handle.

      I saw nothing but hype in your first post and replied to it as such. There is no argument in it. I generally do not follow hyped links unless there has been intelligible reasoning that makes a logical argument supporting the link. "Go here and read this (because the argument is beyond my ability to express)" doesn't do that. If I didn't have that rule, I'd have to read reams of ramblings recommended by any kook on the internet who wants to cite them as "making his case".

      "E=mc^2" has nothing to do with this. Citing it doesn't convert your favored construction into science.

      Try reading Aquinas instead of posting silly mockery of him.

      Tom Cohoe

      Delete
    9. Still stuck in Positivist mode eh Paps?

      Panpsychism-the doctrine or belief that everything material, however small, has an element of individual consciousness.

      That seems like a weird way to rescue materialism. Property dualism would be a better choice.

      Or you can do the sensible thing and admit Materialism is bogus and find a better way to be a metaphysical naturalist.

      Just saying....

      Of course none of this is a problem for common sense types who are rational enough to believe in hylomorphism.

      Again just saying....

      Delete
    10. @ Son of Ya'Kov,

      I set my phone down to slap a fly and when I picked the phone up again, my post was gone way before I was ready. That's what a little fly can do.

      "hylomorphism"

      Now why didn't I think of saying that? It's far simpler and could have been properly posted even before a fly began to bug me. :-)

      Tom Cohoe

      Delete
    11. @Tom Cohoe

      Yer response to Paps is awesome. Well done sir. Be at peace. Winter will soon be here and the bloody flies will die till next spring.

      Cheers.

      PS. Gnus can't seem to move beyond their scientism unlike more rational and educated Atheists and Skeptics. It is why their version of non-belief is so intellectually limiting. If I was gonna go to Hell for being an Atheist(not that I am saying they all go there or that Theists never do) I would at least take care to see I am not stupid as well as damned.

      Why add insult to self inflicted injury?


      God bless.

      Positivism is young earth creationism for infidels.

      Delete
    12. @ Anonymous at 8.56AM and
      @ Anonymous at 11.23AM

      I refer to your two entries at the above times. Both were published by Anonymous. There is no other title in the boxed field reserved for the identity of the publisher of the comment to which it is attached. Equally, repeating the comment doesn't make it any less ignorant. Past history of this blog has demonstrably illustrated innumerable times the deceitful and duplicitous way the commenter, Anonymous, has exploited that anonymity to make comment for which he has taken no responsibility. Simply adding a name, any name, into the content of the comment is no guarantee of your never resorting to unprincipled dissembling whatever message you so choose into the future. If you were an honest chap you would take that one step further and properly incorporate your real identity into the appropriate publisher window. I have done the honest thing from the outset. Linton is my first name. To my grandchildren I actually am Papalinton.

      So, do the honourable thing, get rid of the Anonymous tag. I will be happy to then abide by your request.

      Now, about the matter of E=mc^2; drawing from and repeating what Taylor and Wheeler write about it and your understanding of it is a non-sequitur. Simply referring to what someone else says brings us not a scintilla closer to obviating your profound lack of understanding how science works. So to remove all of the Wheeler and Taylor material in your second commentary is clearly an acknowledgement by you that it was indeed a non-sequitur to affirming your scientific understanding. Be that as it may, I don't hold it against anyone who recognises their own limitations. It is worse still that given our tremendous and continuing growth in understanding and knowledge about us, the world, the universe, many, though fewer every day going forward, still choose to cling desperately to primitive superstition and disembodied supernatural agencies that might or might not do us harm.

      TO BE CONTINUED

      Delete
    13. CONT:

      As a matter of fact, I have read Aquinas. And I can understand why it is that only mostly Catholics extol Thomism as philosophy. THIS ARTICLE, written, believe it or not, by a Catholic philosopher, Dr Brian Kemple, is one of the more honest overviews of Thomism, beset by its enormous internal and unresolved structural problems, even after centuries of argument, unable to reverse its dwindling influence in contemporary society. One such aspect:

      "Among the unfortunate tendencies of Thomists, one deserves special attention: sectarianism. That is, many Thomists tend to characterize themselves as Christian philosophers. It’s all well and fine to be a Christian and to be a philosopher; but to make “Christian” an adjective modifying “philosophy” does suggest that your philosophy is substantially modified by that Christianity, which, I think, undermines the validity of that philosophical and therefore the ability of Thomism as a whole to gain recognition. It gives the impression of appropriating all things which are true under the umbrella of Christianity, or (more frequently) Catholicism, which simultaneously rejects as false all things which are not Christian or Catholic–that is, it turns truth itself into a sectarian divide."

      No better a characterisation than the above by Dr Kempler reflects the Feserian approach to Thomist philosophy today; Christian philosophy in all its sectarian glory and pride.

      Dr Kemple is right here. Thomism can indeed never be seen to enjoy the mantle of universalism that other great philosophical traditions [Aristotelian, Kantian, Cartesian, etc] do, regardless of one's religious baggage. Indeed Thomism is largely a synthesis of Aristotelianism and Christian mythology. Thomism is a catholicised philosophy borrowed from, no less, a theistic pagan philosopher who practiced philosophy well before Jesus was even a twinkle in Mary's eye.

      That was always a fundamental flaw in the bona fides of Thomism as a philosophy. And it is fittingly described here: "Thomas Aquinas was a theologian and a Scholastic philosopher.[88] However, he never considered himself a philosopher, and criticized philosophers, whom he saw as pagans, for always "falling short of the true and proper wisdom to be found in Christian revelation."[89] With this in mind, Thomas did have respect for Aristotle, so much so that in the Summa, he often cites Aristotle simply as "the Philosopher", a designation frequently used at that time. However, Thomas "never compromised Christian doctrine by bringing it into line with current Aristotelianism; rather, he modified and corrected the latter whenever it clashed with Christian belief" [Wiki]

      What may seem an ad hominem to the uninitiated could well simply be a person with a very thin skin and the sensitivity on the tenuous nature of the belief system they are attempting to defend.

      Delete
    14. @Tom Cohoe

      As you can see Paps has no problem boring the stuffing out of us with his tedious Ad Populum fallacy as per usual.

      That is all you will get from him besides the warmed over positivism.

      If yer expecting a rational philosophical defense of reductionist materialism or physical ism yer not gonna get it. Ditto in regards to a genuine philosophical critique of Thomism.

      You are going to get platitudes from this Australia Gnu nothing more. I don't mind people who challenge what I believe but you know I wish they would actually address it rather then give me their touchie feely nonsense.....

      But that is just me. Cheers.

      Delete
    15. @ Papalinton,

      Congratulations on being a grandfather. I advise you however not to teach them to talk to people the way you talk to me. I told you that my name is Tom Cohoe and asked you to use it. Will you teach your grandchildren to assume that people are liars and use that as a justification for being rude to them? I hope for their sake that you do not.


      Here is some reasoning which anyone can follow. "E=m" is a more fundamental expression within special relativity than "E=mc^2" because "c^2" is a constant the numerical value of which can be made 1 by choosing space and time units (they are arbitrary) that work that way. Can you understand that? Refer to Taylor and Wheeler if you want to learn more about it, or don't. I don't care. The reasoning is in my post for anyone to see.


      You have found a Catholic philosopher who denounces Aquinas and quoted him (not Aquinas) in an anti-Aquinas "argument". Whooee! You can find Catholic philosophers or theologians to say anything under the sun, just as you can find people not in the Church to say anything under the sun. That is human nature. I hope you read more than science fiction and "The God Delusion" to learn about these things.


      God loves you and wants to be your friend.


      Best of luck with your grandchildren.


      Tom Cohoe


      PS - I have given my name and that I am Catholic and where I live down to a region of 30,000 people. Check it out instead of insinuating that I am a liar. "Anonymous" is a legitimate method of posting on this blog. OTOH, on this blog, I have seen people change their posting names in non anonymous posts on posts that had aleady been published a week earlier. Nothing guarantees a name on any posting method. Nothing guarantees that "Linton" is your name. Nonetheless, I choose to believe you. Why not extend me the same courtesy, especially since I have even given you a way to physically find me (it's pretty easy - a named Catholic in a region, named in this blog, of 30,000 people).


      Best of luck in your progress. TC

      Delete
    16. Papalinton,

      A 'theory of consciousness described mathematically' is like pointing to sheet music and saying that it 'explains' Beethoven's Ninth Symphony. At best it only captures certain mathematical features of the performance. Everything that's really interesting about it is being ignored or swept under the rug...of the mind, which is where we started with in the first place.

      Delete
    17. @Tom Cohoe:
      Will you teach your grandchildren to assume that people are liars and use that as a justification for being rude to them? I hope for their sake that you do not.

      He's probably going to "teach" (lol) them that they are "purposeless meat-robots" in a "purposeless" Universe and to "think for themselves" though their thoughts "are not even theirs" because they're just causally impotent epiphenomena "created" and manipulated by their amorphous "god" The Laws of Physics.

      And that they're "lucky" to be "alive" though "life" is a "slippery philosophical term" with "probably no objective meaning attached to it". They should "thank" their "mother" (who maybe now "self-identifies" as their "father"?) for "having chosen" not to tore them apart in the womb, though there is precious little "she"(maybe "he"?) could have "chosen" since it's The Laws of Physics who run the show and do all the "choosing".

      And that that all of the above is an objective truth in a Universe where objectivity does not exist.

      Probably will force them to sit through endless re-runs of "I, Robot" and "Planet of the Apes" to "strenghten familiar ties".

      But those are the tenets of the materialist/physicalist religion and he's a faithful adherent. Are they illogical and retarded? Yes. Are they superstitious and laughable? Yes, they are. But "to each brain shaped by the mindless forces of evolution, its own" I guess.

      Delete
  7. If materialism describes the cosmos, it's not much of a cosmos. On the face of it, how would your brain be able to capture that which it is limited by? But then materialists, like madmen, never have doubts.

    ReplyDelete
  8. Good article, although I think if we take this a bit further, we see that the Christian neoplatonists (from Augustine to Scotus etc) were further ahead than all of these.

    Arguably, the leading (sane) theories in quantum mechanics (by which I exclude Many Worlds) state that physical properties only exist in the second person, as part of an interaction (or observation). There simply is not any way in which material properties exist objectively. When you ask the question about the subject and object of the interaction are, they will use a word like ‘agent’ (in the case of QBism). However you’re left with a problem. If these agents have no physical properties in themselves, then what are they?

    Kant had previously come to the conclusion that this ‘thing-in-itself’ was simply unknowable. However Schopenhauer pointed out that in at least one case, we do know something of the subject, because it’s us. So he concluded that there is only mind (although he called it “will”).

    So modern analytical idealism takes these two areas together, and concludes that all is fundamentally mind, and physical matter is just the image of mental processes, a representation across a ‘Markov’/transpersonal boundary. However to achieve this, they use the concept of disassociation, as in DID (multiple personality disorder), and so everything (including the self) becomes like bubbles in the universal mind. This “objective idealism” is clearly an improvement from Berkeley’s subjective idealism, where the consistent world ‘outside’ that we all experience makes little sense, but equally flattens reality, where a cosmic mind behaves instinctively and habitually in terms of the laws of physics, leaving the order and fine tuning of the universe as much of a mystery as it is for the materialist.

    However, if we go back to the church fathers and theologians through the middle ages, up until Ockham and nominalism broke everything, you have a platonic hierarchy of being, with the divine ideas at the top, eternal, flowing down into essences and then forms, like seeds moving towards realisation of the ideas. In this view, modern relational quantum physics makes far better sense, with the material world being the interaction of forms. I’m using “forms” here as the last layer or fold of this hierarchy, that which directly is represented as matter. Even from those more influenced by Aristotle than Plato, it’s only when the modern mindset is retrospectively applied that we end up with this Cartesian interpretation. As Aquinas may put it, the body is in the soul, as it’s outward appearance.

    I’m starting to think that the modern term for the old christian ontology would be octal aspect monism, plus a top three which are absolute and therefore cannot be termed ‘substance’ in a meaningful sense.

    ReplyDelete
    Replies
    1. Greetings Simon. It looks as though you have continued the same heterodox interpretation of St. Augustine you were pursuing on the other thread in our discussion about evolution. I can only repeat that for Augustine and Aquinas, there is a radical distinction between the Creator and creation - No cosmic mind.

      It's no use pushing the undercurrent of heretical interpretations of the Fathers which continued through the Middle Ages and beyond; the fraticelli were not St Francis and Luther was not Augustine.

      Delete
    2. Besides what Miguel pointed out, that there is a ontological distinction between God and creation, i would never call any of the fathers a idealist. The ones who were more philosophically-minded would likely all agree with the common view of the ancient platonists that matter is a real thing.

      Of course, we say matter on the sense of what receives form in a particular way limited in being subjected to space and time*. If one rather defines idealism as "the view that there is no matter as the moderns conceive of it" them most of the classical and medievals are idealists, i guess that only the atomists would not fit. Our definition of matter mat... is important if we want to know if it exists.


      *not that these are actual beings, Aristotle and the boys were very smart and knew than that does not work

      Delete

    3. Hi Miguel

      You completely misunderstand me. I’m in no way at all supporting pantheism. You are assuming a cartesian understanding of hylomorphism, as if matter can exist all by itself, which is NOT what Aquinas believed. There is a basic understanding of things in the early church which developed to understand things like the trinity and creation ex nihilo, which has been confused to the modern mind by the development of cartesian dualism (preceded by the error if nominalism, and followed by the heretical pantheism of the german idealists). This older way of thinking is embedded in our Catholic faith, even in the Nicene creed. If you use the modern understanding of realism, with matter at the pinnacle of reality, then it is not a christian ontology you have.

      Instead, “the locus of existence is neither matter nor the form that enters it, since the former is no-thing without formal properties and the latter is incapable of existing on its own. Form only comes to be when manifest in matter, but even then it requires a concrete individual, or hypostasis,in which to subsist. The subject is thus anterior to both form and matter, constituting the locus of existence in the hylomorphic subject.
      […]The above description is true of all creatures, including “immaterial” ones, such as angels and souls. For even these, though “immaterial” in comparison with flesh, are nonetheless hylomorphic and subject to generation, change, and corruption. The only true exceptions are the divine hypostases themselves, Father, Son, and Holy Spirit. For these, it remains true that the hypostasis is the locus of existence and that the nature is common and undivided. But these divine
      hypostases alone are truly immaterial, being immune to becoming, mutation, and corruption.”

      Similarly with the two natures of Christ;

      “Though his natures are two, and though both natures include capacities of volition, for example, only one hypostasis ever uses the-
      se natures; thus, there is no danger of disharmony, as if Christ might be divided against himself. For there is only ever one subject using the natures in any given act. Likewise, in disputes concerning the two energies of Christ—referring to distinct operative powers of Christ’s human nature and his divine nature—the Eastern fathers introduce the theandric (God-man) operations, which do not deny, conflate, or confuse the operative powers (energeiai) of either of Christ’s natures, but
      rather suggest that these distinct natural energies always operate l harmoniously and in tandem because only one hypostasis uses both. John of Damascus illustrates the
      point with an enflamed blade, which has two sets of operations (to cut and to burn) from two different natures (blade and fire). Yet, because the blade is only one subject, both operations are present and harmonious in the singular act of the blade:
       When run across a surface, it cuts and burns simultaneously.”

      The same article goes on that for the Fathers…;

      “the
      logoi certainly include forms, or universals, and thus model the cosmic
      system of genera and species—the so-called “Great Chain of Being.” Yet, unlike
      pagan realism, which elevates Form, while denigrating matter and ignoring individuals, the Eastern fathers include individuals amongst the works of God that articulate divine wisdom. Hence, the logoi
      include “universals as well as particulars.”
       
      In other words, the Eastern fathers understand the logoi to be models for all that God creates. Hence, they are not limited to universals. God creates universals and
      individuals; he is the creator of matter, and is the God of history.”

      Rather than quoting any more, I suggest you read the paper these come from. It focuses on the Eastern Fathers, but as several Popes have affirmed, these are not in conflict with the Catholic church in these matters;

      https://www.academia.edu/41586437/The_Metaphysical_Idealism_of_the_Eastern_Church_Fathers

      Delete
    4. Hi Miguel

      Perhaps my original reply was too long, so I will try a shorter answer. It’s clear that you completely misunderstand me. I’m in no way at all supporting pantheism. You are assuming a cartesian understanding of hylomorphism, as if matter can exist all by itself, which is NOT what Aquinas believed. There is a basic understanding of things in the early church which developed to understand things like the trinity and creation ex nihilo, which has been confused to the modern mind by the development of cartesian dualism (preceded by the error of nominalism, and followed by the heretical pantheism of the german idealists). This older way of thinking is embedded in our Catholic faith. You seem to have a cartesian dualist ontology, which is NOT the Catholic view. Under this position you end up with Christ having two separate natures etc.

      This may help;

      “This line of thought the Scholastics adopted, developing it in their treatises as ideology. Their theory is described not as idealism, but as realism; but this does not imply that they are in conflict with the doctrine of Augustine; it means rather that the ideal principles possess real validity, that as ideas they subsist in the Divine mind before the things corresponding to them are called into existence, while, as forms and essences, they really exist in nature and are not really products of our thinking.”

      https://www.newadvent.org/cathen/07634a.htm

      Delete
    5. @Talmid. I agree that it depends on your definition, and the German Idealists hijacked the older more platonic sense, and replaced that with pantheism where creation = god. In the older sense mind and body are not distinct in a cartesian way, and it’s certainly not a reduction to matter.

      Delete
    6. Seeing you quoting it, it is a cool position. The confusion is thanks to the use of "idealism", who makes us think of very heterodox ideas.

      Since the germans, it seems that anyone who calls himself a idealist is defining his position by confronting the modern sense of matter, so, it tends to not work even while it correctly rejects the modern matter. That is why "idealism" needs to be used today with a bit of explaning.

      Delete
    7. “That is why "idealism" needs to be used today with a bit of explaining. ”

      Yes I agree completely. However I do think it’s valid to use it, given that when we talk of the things of the world, most people don’t really think about what any Christian before Occam actually believed. We use terms like “dual aspect”, hylomorphism, neoplatonic, Aristotelean, but in peoples mind they are usually thinking about cartesian dualism. This weirdly (especially in places influenced by certain types of protestantism) end up as a kind of materialist physicalism, with god bolted on the top! Some then go “spiritualist”/new age, and adopt a gnostic distortion of this.

      We really need to rediscover what a christian ontology by definition means. You see it in church docs, John Paul II encyclicals such as Fides et Ratio, and in philosophers like Jean Borella. It also of course goes back all through the scholastics to the Church Fathers and Paul etc, but I often see Thomism used as if he was closer to the cartesian materialism with god bolted on. If god is there, who am I to complain, as that is the most important part of any true ontology, and none will be absolutely true in this life. But I do think it causes problems, such as a literalist reading of scripture, and a thinking of the ‘spiritual world’ as separate from us…

      Delete
    8. Greetings Simon. I you are able to find Cartesianism in my last comment, you will have produced a true miracle for this thread.
      The Fathers of the Church were all Catholics and you should have quoted them instead of that article by two completely unorthodox writers. One of them, Benedikt Gocke, defends “panentheism”. Playing around with this (and making up one’s own religion in the process) gets us no further away from errors like pantheism.
      In Gocke’s version, God “needs” the world, in relation to which He is both dependent and influenced. This makes a mockery of orthodox Christianity, which insists on God immutability and impassibility. The list of those who have supported panentheist-type ideas reads like a rogues gallery of heretics: Eckhart, Boehme, Bruno, Spinoza, Krause, Hegel. Awful.
      If you support panentheism, then it’s easy to understand why you can’t accept “creation” or any divine action happening any way apart from evolution. It’s a little disappointing, as I thought I had been arguing with an orthodox Catholic who just happened to be particularly convinced by the science. Instead, it’s just the boring history of the theory itself again; philosophical evolutionism preceded biological evolution, which was seized upon, and has remained interested and contrived in that manner. I retain a healthy doubt about it all.
      Panentheism isn’t religion, and doesn’t deal with God. It continues to use religious terms and discuss “religious” themes, but this isn’t what the Apostles or the Fathers of the Church preached. By emptying terms of their real meaning, errors like this lead people away from the faith, and fool those who look for it to remain content indefinitely with this playground of verbiage that hides true religion.

      I don't know if I really want to talk about the merits of each one of the galaxy of heterodox writers. Could you make it simpler by using indisputably orthodox Catholic sources?

      Delete
    9. Hi Miguel

      I focussing on modern terms may not help from your perspective. There was a time when I thought that panentheism was a good modern term to use, but I don’t feel comfortable at all with it. It tends to mix god with creation far too much the way most people use it. Of course Paul did say “In Him we live and move and have our being” in his speech in Acts, but most panentheists would take this to be describing the world as god, which I would agree with you is not Catholic.

      However we should be careful here, because there are weak forms of panentheism which are fully Catholic. What the Eastern Orthodox church refer to as god’s energies, we usually refer to as grace. We fully accept that god can be immanent. When I go to my hour in Eucharistic Adoration, there are times when you know very clearly that god is present. Of course we know that he is present in the eucharist, but this is an awareness of god overflowing into a place and time. The strong pantheist would say that the only difference from when I am, say, sitting in my car or in a field, is that I am focussing my mind on god, and that he is present everywhere all the time. There is a way in which this is true, everything was created by him and everything is sustained in being by him, he knows every hair on our head, and wherever we are, we can turn our mind to him in prayer, and he hears. However his active presence is quite different to this, especially in those like St John of the Cross or Theresa of Avilla where this experience of his presence was lifted to a conscious joining, what we usually call theosis, or divinisation. Those of us less holy, can still experience god anywhere, but apart from the time when I first discovered him, for me at least it’s rare other than in eucharistic adoration. I don’t understand whether this is a different person of god from when you speak about him and feel a tremble, but the fact is he comes in, and then goes. So god exists outside of time and space, but becomes immanent in a time and space, to an individual. So he is aware of everything in his transcendent nature, but he is only immanent at particular times and places, so certainly that rules out a strong panentheism.

      However the question of god’s presence is different from the question of the substance of the universe. If you want to stick with Aquinas, we have the substantial form plus prime matter. So the closest we have seen in physics to prime matter is the quantum field, where all particles are excitations of this field. For the particles themselves, Heisenberg considered these to be forms, or ideas. Whichever catholic ontology you choose, be it Platonic or Aristotelean, there is a natural sense in which the substrate of creation in ‘like empty mind’ (prime matter to Aristotle/Aquinas etc, prime space to Plato) and the content is like thought. I’m certainly not suggesting creation is god’s mind, because that would be pantheism. Also nature behaves habitually, instinctively, as the ‘laws of nature’, and god is not a creature like this. But I assume you accept that your mind is not separate from the matter in your body? If so, of what substance are you made?

      Delete
    10. Also there are many other senses in which you will run into problems with basic catholic theology if you do not accept that creation is essentially’mind-like’. For example, all the great catholic theologians before nominalism reared it’s evil head, believed in universals. To accept these, you have to accept that creation works like the human mind, that there is a reality to these beyond our perception and communication of them. If not, what are universals to you?

      Delete
    11. Greetings Simon. We can’t accept the Orthodox and the “divine energies” mysticism popular among them as fully Catholic. This old article from the Catholic Encyclopedia is still a valid description of that phenomenon. https://www.newadvent.org/cathen/07301a.htm

      Certainly the presence of God is real, but does not diminish the absolute distinction that exists between him and the universe. Nor is He the “mind” or “soul” of creation. The existence of universals in no way impedes God's complete freedom to create; the world is not "necessary" to him. There is no comparison between the relation between our mind and our bodies and that of God with creation, unless one adopts Gocke’s notions. His ideas still use terms like God, creation etc., but plainly don’t mean the same thing by them as religion intends. These kinds of ideas don’t raise nature up to the divine, they debase the divine. Most often such notions are held by people who don’t believe in God. Not all of creation follows the laws of nature instinctively and that is another argument against identifying God with nature.
      It's obvious that we are not divine. I can’t understand why people invent ideologies to convince themselves of this. Believing and practicing the Faith is already a great thing.

      Delete
    12. Hi Miguel

      I think I can see why you keep responding to a complete straw man of what I am saying. In effect you make the same false assumption that the German idealists make.

      Let’s go step by step. First I suggested that as our bodies and minds are of the same substantial form, they must by definition be of the same substance. You keep assuming that the body and mind are separate metaphysical substances, and then denying that you are heavily influenced by cartesian thinking.

      Now step 2, and without bringing god into the picture at this stage, it’s natural to extend this to the rest of creation. If we are of a “substantial form” substance, then it strongly suggests that the world we interact with is also the same substance. Note that I am not in any way suggesting that living substantial forms are the same as inanimate ones, we’re just talking about substance.

      Step 3, I am saying that is our bodies and minds are composed of the sane substance, then it’s incoherent to reduce that to purely material properties. Physicalists have been trying to find ways to get conscious experience from matter for decades, and the likes of ex-materialists behind IIT embracing panpsychism (equally incoherently) is evidence that there is just no way to get experience from matter (despite the continuing delusion that there will eventually be some magical breakthrough). So even using a basic scholastic Catholic ontology, this substance seems more like mind (or maybe) than anything we have a word for. You could maybe use “spirit”, but that’s arguably semantics in this context. I also used relational quantum mechanics interpretations to support this view.

      Step 4. Now you suddenly jump to the conclusion, that because I’m saying that the substance of creation is closer to ‘mind’ than any other term we have, that I am saying that creation is made OF god’s mind. That IS pantheism, and I am NOT suggesting that at all. When the bread is blessed at mass, it is one substance, and then it takes on the substance of Jesus. So I would be crazy to be a catholic who believed that god somehow replaced himself with himself!

      With the universals, you surely do have a problem from your perspective, and it was people like Occam who first didn’t see the substance of the universe as ‘like mind’ who naturally adopted nominalism. Our minds create universals as a natural part of engaging with creation. For this to be real, we must be re-representing something already in creation (unless of course it’s something we have created). In other words, the forms in creation are grouped by essence in the same way our minds group them. So their is a natural hierarchy of being, and parts of that hierarchy are intelligible to us because our minds are not different to it. More than this, we know that it’s more than just a coincidence, because when we use the mathematical universals in equations, we can predict new things about the universe. So these universals are embedded deeply into creation.

      I agree that our faith is a great thing, and the last thing I want to do is challenge it. For me the truth of our faith is self evident. I’m not trying to invent a new ideology, I’m trying to keep it safe from the damage still being done by nominalism, Decartes, relativism and modern philosophy in general.

      Delete
    13. Also I’m curious what it means to you when scripture says, for example, “sun and moon bow down before you”? In this case it’s clear someone like St Francis of Assisi took it one way, but I suspect you will have a different view to him?

      Delete
    14. Greetings Simon. No Cartesianism here. Substance can have different meanings. In response to you points: 1 - It is one thing to say body and soul are one substance, and another to say they are of the same substance. 2 - The world we interact with is composed of other substances. 3 – Our bodies and souls are not composed of the same “substance”. Souls are immaterial by definition. 4 – If you think there is a cosmic mind of the kind you mentioned in your earlier comment, even if it is not God, you are still wrong.
      There is no support for the notions you have put forward from the Faith or from Thomism. I’m a bit bemused at being scolded for not agreeing with your ideas when they are so odd and you have used heterodox writers to back them up.
      It would be pompous for me to try to pontificate on the inns and outs of hylomorphism. Surely you are aware of the basics as scholastics understand them. I’m just a Catholic peasant guerilla with a pitchfork laying in wait for the ideologies Mater Ecclesia has so clearly told us to attack.

      Delete
    15. Oops. Lying in wait. Us peasants are careless about some things.

      Delete
    16. Haha … there was certainly no scolding intended from my perspective!

      The notions I’m putting forward are arguably closer to Bonaventure than Aquinas, but if there is anything I am saying that conflicts with our tradition or with scripture (rather than just badly phrased through writing via my phone in the middle of the night) then I want to know. When I used the phrase “cosmic mind” I was specifically referring to modern idealism. The early church writers were naturally cautious about using Plato’s “world soul” concept, but never in Plato or the church fathers would this be equated with god.

      Also of course metaphysical substance is not the same as the way substance is used in science. This causes a lot of confusion, and comedy moments like the study a little while back when chemists “proved” that the Host does not change substance when it is blessed. Up until 700 years ago, your distinction between material and immaterial could be also be seen as a distinction between inward and outward. This inward reality has been reduced to something like a vapour, an illusory space in which to process, and the ‘real world’ is the ‘objective world’ of the senses. However the science of the last century has shown that there is no objective world, it’s all observer dependent relations. So with the inward reality reduced to illusion, the outward reality essentially relative, and god removed from the picture, it’s all reduced to absurdity. The world is going mad as a result, and we as catholics are too caught up in the madness to do anything about it. We’re turned into a fortress under attack, and yes we all have our pitchforks out! We are indeed under attack, and there are real forces doing what they can to discredit the faith, and yes, to introduce errors. However those errors are 700 years old, first nominalism, Luther, Descartes, protestant scriptural skepticism, then a reaction against those in terms of pantheist idealism, a reaction against those in terms of positivism, a reaction against that in terms of post modernism. It’s the landscape we walk in. One way to avoid that is to box everything up, and keep the faith in a secure and well alarmed box, but then we don’t really see how these modern ways of thinking affect us in each of these boxes.

      Anyway I’m rambling on again!

      Delete
    17. Greetings Simon. It's difficult to discuss things with you as you don't reply to the points that are made, and bring up more heterodox ideas. Perhaps you should put forward your position on things generally and explain why the orthodox Christian and scholastic approach is wrong.

      Now you've provided another pretty awful modern idea; that modern science has shown there is no objective world. There is nothing further from St. Augustine and the Fathers, or St Bonaventure and St. Thomas. Like your insistence that Adam was not the first man, or that there is a cosmic mind, identified with God or not, it's contradicted by Catholic orthodoxy.

      Catholics who know their faith don't go mad. I can't say the same for those who don't know there is an objective reality.

      I dispute the version of modern history as a tale of the Church being on the back foot, boxed up and generally losing touch with the march of events. The Renaissance was a foretaste of the errors that are hegemonic today. Yet the Renaissance was defeated by the Church in the shape of Baroque civilisation. It was the secularist and evolutionist ideology that was boxed up in northern Europe by the global hegemony of the Baroque that lasted over a century. In the end, Baroque hegemony was not brought to ends by its inability to handle false ideologies (they simply failed to make much impression on these countries after the defeat of the Renaissance) but by military defeats brought about by an alliance of the "Third World" of the time - some Protestant countries, the Turks and France. The power of war, not ideas, Simon. This war isn't over as long as the Church is still standing. We are very fortunate to be Catholics because we have a coherence that out enemies can never have. The weakness of Catholics and the Church itself has come about by taking false ideologies seriously. This is not due to any validity on their part that allow them to impose themselves, but on human faults on our part, as history has shown. These ideologies were defeated by reason long ago. Because false ideologies appeal to the worst in human nature, their final defeat is not sealed however until they are shouted down. Pitchforks are not superfluous or decorative either.

      Delete
    18. Hi Miguel

      We keep talking past each other. I do not believe that the “the orthodox Christian and scholastic approach is wrong”. Quite the opposite.

      I’m not sure what points I have ignored, but where you have just missed my point, I have tried another approach - clearly unsuccessfully! The great thing about our faith is that it works just fine without needing to understand how creation is woven together. Nonetheless, there is nothing “heterodox” from a catholic perspective in trying to understand secondary causes better, and I remain confident that the mechanistic, cartesian ways of thinking has influenced the way many of us understand what our own faith already says. As such I will have one last try to explain where I am coming from….
      _____

      How do we make sense of the world? There is certainly a way in which we are pattern finders. It’s reasonable to assume that any group of people would naturally group and name things with similar patterns, say dogs, cats, rivers or trees. Up until around 700 years ago, the common view was that this process was a partial recognition of something embedded deeper within nature than the individual occurrences of these patterns. So apart from the existence of individual apples, there was a hierarchy of being where the very idea of an apple was something real in nature. The terms used for these template ideas was universals. This can sound strange to us now, but it was based on a view where nature was more integrated. The mind and body were not separate parts of a person, but the same person looking inward or outward.

      This way of thinking had several major changes to where we are now. The first major change was from an English friar, William of Ockham. His view was simply that these universals have no real existence in nature, that they only existed in the human mind. This lead to a fierce intellectual war within universities. The catholic church never accepted nominalism, recognising that it conflicted fundamentally with fifteen centuries of it’s theological tradition. However within universities it won the day, and because it had such a big influence on the likes of Martin Luther, it found it’s way back into christianity. So the last few centuries have been heavily influenced by nominalism, in the sciences, in philosophy, and also via protestantism. The old understandings have not been lost completely, particularly in areas where the catholic church continued to dominate. In many ways, this is why we have “continental philosophy”, as opposed to the philosophy of the english speaking world where the protestant ideologies were dominant. Nonetheless, because of the dominance of nominalist derived assumptions in philosophy, the Catholic church adopted Aquinas as the standard by which to refute the errors that quickly became fissile.

      Stepping back the tides of history, it’s clear that Nominalism had a major impact. The church had experienced many heresies in it’s history, but these were argued and debated within a context where god and creation had a connection with human reason. In fact this way of thinking reached back to Socratic times. So drastic was this change, that we can speculate about whether it was prophesied in scripture. God, being outside time, made the Old Testament as a reflection of the New Testament. The Ark of the Covenant was Mary, Noah’s Ark is the Church etc. Perhaps the scholastics were the Towel of Babel, reaching close to the edge of heaven, and Nominalism the confounding of man’s understanding.


      Delete
    19. I won't talk past you. This is why you'll need to explain why your assertions that Adam was not the first man, that there is no objective reality, and that there is a cosmic mind, are not a complete contradiction of Catholic doctrine and scholasticism before the conversation goes any further.

      Delete
    20. Morning Miguel

      I do not claim that there is “a cosmic mind”, I used this term when describing modern pantheistic idealism. What I was trying to explain to you is an aspect of thinking that was the default up until 700 years ago. Read Bonaventure “The Journey of the Mind into God”. In this there is a continuous movement up Jacob’s Ladder, with the first step being the external world, and then the next steps are inward. This hierarchy of being from earth to heaven was the natural understanding then. The mystery of the bread becoming the Body was Jesus transcending this hierarchy, heaven and earth touching in the Host. It’s a mystery of course, but it’s connected to Jesus as the ‘ladder’ reconciling heaven and earth. It’s not one physical world, and a separate, disconnected physical world, as modern man thinks.

      In terms of objective reality, there is of course a consistent, external and real world. What I am explaining is that we are not separate from it, we are part of it. There is not a separate external world that has one third person perspective. Time moves at a different speed for someone flying in a plane than it does for someone on the ground, the length of objects changes depending on how fast you are moving, the particles that make up the physical world do not have properties until they ‘entangled’ with an ‘observer’, and they can have different properties to different observers. This is basic, standard physics, and does not conflict with catholic teaching.

      In terms of Adam, we have already discussed this on the other thread. You choose to have a literalist interpretation of this, as protestants and many modern people do (especially in the US), but this is not required by the church. The church is comfortable to say that Adam was a real historical person, our spiritual ‘first parent’ created by god, and to leave conclusions about the secondary causes through which god achieved this act of creation open to debate. This is clear from statements by recent popes. I assume that you don’t have a literalist interpretation when scripture says things like “Praise him, sun and moon; praise him, all you shining stars”, you choose to do so depending on your preconceived assumptions, and that’s fine as we all do that to some extent. But you can’t claim that your own assumed modernist views are the only orthodox catholic views, especially when our tradition is not a literalist tradition.

      Delete
    21. Greetings Simon. Your reference to a cosmic mind was in your first comment here. You weren't talking about pantheism. You wrote that the laws of physics obeyed a cosmic mind. On objective reality you wrote that "the science of the last century has shown that there is no objective world, it’s all observer dependent relations". Nothing in your last comment deals with with that assertion. Finally, in Humani Generis, Pope Pius XII states definitively that Adam being the first man is to be understood in the literal sense. Your interpretation is not orthodox. You have to change it.

      I'll really have to leave it here though. Some interesting issues all in all.

      Delete
    22. Hi Miguel. Yes I mentioned modern analytical idealism, but I didn’t say that I agree with it. In fact I specifically said that it flattens reality, looses the vertical - the whole point I was trying to make.

      Anyway I agree that there is no point continuing these circles. I will leave you with two quotes;

      First Aquinas;

      “On the day on which God created the heaven and the earth, He created also every plant of the field, not, indeed, actually, but “before it sprung up in the earth,” that is, potentially.…All things were not distinguished and adorned together, not from a want of power on God’s part, as requiring time in which to work, but that due order might be observed in the instituting of the world.”

      And Augustine;

      “ In matters that are so obscure and far beyond our vision, we find in Holy Scripture passages which can be interpreted in very different ways without prejudice to the faith we have received. In such cases, we should not rush in headlong and so firmly take our stand on one side that, if further progress in the search of truth justly undermines this position, we too fall with it. That would be to battle not for the teaching of Holy Scripture but for our own, wishing its teaching to conform to ours, whereas we ought to wish ours to conform to that of Sacred Scripture.”

      Delete
  9. "The sober, boring truth – enshrined in Aristotelian philosophy and common sense alike – is that some kinds of purely material substances (namely non-human animals) are conscious, and others (like rocks and dirt) are not."

    Gilson: "What separates us irreparably from [modern science] is the Aristotelian (and common sense) notion of Substantial Form ... Descartes rid nature of it. They understand nothing anymore since they forgot Aristotle's great saying that 'there is no part of an animal which is purely material or purely immaterial.'"

    ReplyDelete
    Replies
    1. One of the difficulties in writing about these topics is that the key terms (such as, in this case, "matter" and "material") don't have the same meanings and connotations in discussions among contemporary analytic philosophers that they do in discussions among Aristotelian and Thomistic philosophers. Sometimes, for the sake of readability, I'll use such terms in their modern senses (as I did in the passage you quote). But it would have been better had I referred instead to "purely corporeal substances." As you rightly note, in the Aristotelian sense of "material," not even purely corporeal substances are purely material insofar as they always have form in addition to matter.

      Delete
    2. To add to what Feser said: the forms of physical substances, while not being matter (in the Aristotelian sense of the term), are themselves dependent on matter to exist. They can thus be called 'material' in that sense.

      Delete
  10. Are most contemporary philosophers of mind not dualists?

    ReplyDelete
    Replies
    1. I mean materialists.

      Delete
    2. No, most are still physicalist monists, although the portion of others is increasing.

      Delete
    3. I think there is some reason to say that property dualism is as popular as pure physicalism amongst contemporary philosophers. Although most modern philosophers the former in as materialist a way as possible (mental properties and states have no causal power, etc.,), it technically is not materialism.

      Delete
  11. "But that they are in the mind rather than in the body is precisely what post-Cartesian philosophers, whether dualist or materialist, do say."

    I doubt many people (even philosophers) would say the warmth in the throat that results from a swig of hot coffee is in the mind, or that a twinge of pain in the knee or lower back, etc, etc, are in the mind. If I went to a doctor and said I had a pain in my knee, he would examine my knee, not my mind. If his diagnosis was, "that pain is all in your mind," I would look for a competent doctor.

    My assumption is that the mind itself is an experiential machine. It's designed to collect and analyze sensation. How would that data arrive? How would we know about it if not through qualia? It has to arrive as something. And that something would always be a "qualia" of some sort if we are made consciously aware of it.

    It is no conflict to believe that only matter can sense matter, that transmission of sensation is material, and that analysis of sensation is material. Qualia (or a secondary quality) is not a problem. That's just the mechanics of our sensory organs. To claim that a materialist moves a "model" of reality into the mind is not a problem either -- because the materialist assumes the mind is a material machine designed to do just that. To destroy that assumption one must prove that a material mind cannot model the physical world, that "representations in the intellect" cannot be 100% material, and this is not going to be possible to prove.

    IMO, consciousness is the only real problem. And qualia/experience has nothing to say about that problem.

    ReplyDelete
    Replies
    1. No Don, you’re exactly wrong, that’s precisely what philosophers DO say. Of course, normal people don’t talk like that, but that isn’t the point. The point is this: in some philosophies, matter has only numerical properties, but “pain” is a quality not quantity. The problem of consciousness is only a problem because of qualia, so don’t say that one is a problem but not the other because that just misses the point or begs the question. Further, you cannot assume that the matter can somehow sense or have experience and claim victory, because given other materialist assumptions that is simply incoherent. To say “matter (in materialist terms) can sense matter” is to say “something which we have defined as not containing anything but numerical properties can contain some non-numerical property”.

      Delete
    2. If I understand you correctly, you're saying that consciousness itself is the heart of the hard problem (HP) as opposed to qualia. Once you have a conscious/aware machine like the mind which collects and analyzes sensations, qualia is a necessary part of how that machine processes information.

      Some thoughts:


      Can we actually form a useful distinction between consciouness and qualia the way you suggest? The illustrious journal of Wikipedia provides the definition I'm most familiar with: "In philosophy and certain models of psychology, qualia...are defined as individual instances of subjective, conscious experience." I can't even in principle imagine what consciousness could be apart from qualia, if qualia is just that which consciousness is made of. So this suggestion to "assume consciousness, then see that qualia follows," appears vacuous. Unless we can properly distinguish consciousness and qualia, it just sounds like the following: "if we assume the existence of the thing whose existence we're trying to explain, we no longer find ourselves with anything left to explain." In short, I completely agree that if an information machine is conscious, it will operate in terms of qualia....but I fail to see what insight this provides.

      In your second paragraph, after claiming we can eliminate qualia as a distraction and then implying another questionable distinction between qualia and "models" (what exactly is the model we're working with in our minds if not another instance of conscious experience, i.e. qualia??), we are still left with just assuming that the "mind" is both "an experiential machine" (i.e., one that generates experiences) and that it is entirely material.
      So long as we can't 'destroy that assumption' (presumably by demonstrating a contradiction in logic or with empirical data), we're free to run with it.

      People like myself who consider the HP to be a real one would want an actual explanation of how the mind generates experience. I think that's a valid question to investigate and not one I'm content to merely settle as a brute fact that doesn't entail any contradictions (assuming that it doesn't).


      In comsmological arguments about the most general features of reality, I'm more sympathetic to the "brute fact" approach -- as an agnostic it seems plausible that the fundamental basis of reality would in some sense escape our explanation. But something as small, particular, contingent, and cosmically isolated as human consciousness?? That seems extremely counterintuitive. Why be contented with leaving this one particular physical system within a vast cosmos as a mere "given"?

      But if I'm wildly off base and you aren't suggesting we can simply assume consciousness as a given, then we must be able to provide an explanation for it. As a believer in HP, this precisely what I want but see a remarkable absence of in every single materialist account I've found.

      Delete
    3. I just visited your blog where the first result on the Mary's Room thought experiment includes the following:

      "Therefore the brain must have some aspect that depends on a non-physical 'substance' that allows for this experience. Philosophers call this 'qualia.'"

      Qualia is the "non-physical substance" that allows for conscious experience? This is simply not what the word means. I have checked the SEP and see it provides a similar definition as in Wikipedia: https://plato.stanford.edu/entries/qualia/

      Not even Daniel Dennett defines qualia in this way.


      Without wanting to be condescending, I really think your comment boils down to a simple mistake over definitions.

      Delete
    4. Journey516,

      "Further, you cannot assume that the matter can somehow sense or have experience and claim victory, because given other materialist assumptions that is simply incoherent."

      That's overly broad. Materialists don't all have the same assumptions. A charge of incoherence may apply to some but their incoherence is not really my problem. Just to clarify, I am not proclaiming victory. I say that certain lines of reasoning do not refute my materialism. If "in some philosophies, matter has only numerical properties," I'm not obligated to defend those philosophies. For example, I say numerical properties are descriptive. Descriptions are not the things themselves. I would never argue matter contains "numerical properties" -- the implication being that matter is a collection of numerical properties. So the incoherency of “something which we have defined as not containing anything but numerical properties can contain some non-numerical property” does not apply to me. Could you reword this into something I see the need for *my* materialism to refute?

      Delete
    5. Anonymous,

      "Can we actually form a useful distinction between consciousness and qualia the way you suggest?"

      We can "listen" to music in our head long after the actual sound has hit our ears. Does this internal playback qualify as qualia? I don't think so. There is something fundamentally different between experiencing and remembering.

      A brain in a jar is incapable of external sensation. But is it still capable of consciousness? Does a stream of qualia after birth actually create or switch on consciousness?

      As a firmware engineer, this is the way I look at it. A microcontroller can have various sensors attached: maybe an accelerometer, a magnetometer, a thermometer. A protocol allows the microcontroller to query the sensors, then the microcontroller processes that data. These are distinct things: sensing, communicating, processing. Qualia for me is equivalent to the communication protocol. Qualia is like an "agreement" between microcontroller and sensor in how they exchange data (this is analogy since the micro does not have qualia). Qualia is a means to an end, not the end. Qualia (communication) is a functionally necessary part of how the conscious machine receives data, but not in how it processes the data. So I do think there is an important distinction.

      "People like myself who consider the HP to be a real one would want an actual explanation of how the mind generates experience."

      To elaborate on what I said above, I disagree the mind generates experience. It analyzes experience. It makes decisions based on experience. It's even modified by experience. But it is not a generator of the experience itself (although a hedonist might push that boundary).

      I do not consider consciousness to be a brute fact. It has not yet been explained, but I think it will be. I also think it will be replicated in machines -- an unpopular opinion in these parts. To me a dualist 'explanation' for consciousness is inherently a brute fact.

      However, like you, I can't imagine what consciousness would be apart from qualia. What would we think about? Maybe like that old song Peggy Lee made famous, "Is That All There Is?"


      RE: that blog entry:

      You ask, "Qualia is the 'non-physical substance' that allows for conscious experience?"

      No, that was not my meaning. I'll add one word that should clarify: "Philosophers call this *experience* 'qualia.'"


      Delete
    6. Thank you for the clarifying reply, Don. Some more thoughts (apologies for the length):

      1.) With the music and the brain-in-a-vat (BIV) examples, I take your definition of qualia to just be sensory input (or more precisely, the "first-person" impression of sensory input). This seems to be how you distinguish experiencing vs. remembering: the latter case is not predicated on (current) sensory input.

      Whereas I would absolutely insist that the "internal playback" of listening to a song in our heads is still an instance of qualia (on the Wikipedia/SEP definition I take to be predominant). It doesn't make sense for us to talk about the "remembering" of the song without also talking about the experience of *what it is like* to conjure that memory into the forefront of our awareness and experience it there. Since remembering and playing a song in my head is an instance of conscious experience, I call it "qualia." So gain, the idea of "assume consciousness, then see that qualia follows" appears vacuous.

      If we interpret "qualia" in your arguments to mean "impressions of sensory data," then I just take your point to be that the notion of "consciousness" is more fundamental than sense impressions. Okay, fair enough: assuming that it's correct, maybe this distinction helps us understand what consciousness is(n't) by way of negation. I'm still unsure how this really helps to answer the HP, which deals with the existence of qualia as I defined the term.

      2.) You aren't saying the mind generates experience, just analyzes it? Sure, I'll grant that definition of the mind. But that leaves how matter and energy generate consciousness, which I take to be the HP in a nutshell.

      Whereas you think a materialist explanation of consciousness is forthcoming, I as an "HP-er" lack that confidence. I base that opinion on:

      2a.) The complete lack of progress among materialists of various stripes (whose work I've encountered) to actually work toward this end. I'm not a professional, so maybe someone in the field is actually attempting it, but nothing I've seen from the big hitters has fit the bill for me.

      2b.) Arguments I find convincing which suggest a material explanation cannot even in principle account for conscious experience. You've likely heard these a million times but we don't need to rehash them here.

      3.) Re: brute facts -- these very well might be unavoidable regardless of what metaphysical system you adopt. I'm not saying they are or aren't, but if our hands are forced we should always prefer to adopt them "sooner" rather than "later" in our ontology. This is much like how mathematicians adopt their most basic, general axioms at the outset and reason from there. (I understand the purported difference between a brute fact and one that entails its own explanation, but I don't think that's relevant for my example).

      If, halfway through a mathematics textbook, I find that an intricate proof using the text's initial axioms and previously demonstrated results fails to establish a theorem fundamental to the rest of the text, I'm going to say one of two things: either provide a satisfactory proof using what you've shown thus far, or add the simplest axiom(s) you need to prove the theorem.

      For a physics example, consider how physicists have adopted certain things into their ontology when necessary. If we have to accept some aspect of consciousness as basic, I don't seem that as categorically different from taking space-time to be basic. It carries a big ontological price tag to be sure, but you do what you've gotta do!

      Delete
    7. Hi Don -- I think I may have just replied to "Neverthelessly" right below your comment...if that's the case, rather than reposting my diatribe in full I will just ask you to refer to that comment thread instead.

      I'll make a blogger account to avoid mistakes in the future. Best regards.

      Delete
    8. " [...] I disagree the mind generates experience. It analyzes experience [...]"

      Perhaps you have heard the old computer dictum "Garbage in, garbage out - GIGO".

      You can't magically rise above invention without assuming the magical end. But garbage just stays garbage. You are besotted with computation as comprehensive of reality when computation is just an image of reality.

      The world is bigger and more amazing than any product of imagination.

      Tom Cohoe

      Delete
    9. @Don Jindra
      “ I do not consider consciousness to be a brute fact. It has not yet been explained, but I think it will be. I also think it will be replicated in machines -- an unpopular opinion in these parts.”

      One think to think about here. Any computer is equivalent to plumbing. You have pipes with water flowing in them, and various types of joints and taps/valves. How big would this plumbing have to be for it to have conscious experience?

      There is no technology we have at present that I can even imagine becoming aware. It’s an assumption persistent in our culture, but nothing at all to explain where it would come from other than an assumption that our brains are basically like computers…

      Delete
    10. Don Jindra,

      So you say that the mind is an 'experiential machine' and that qualia are the means through which we come to know data. How are you defining 'data' here?

      In your analogy a microcontroller 'queries sensors and processes data', yet with the caveat that the micro does not have qualia. If that's the case, why is any qualia necessary at all? Are you claiming that it's a 'spandrel', a byproduct of something that has utility? Or that it has utility in itself?

      Delete
    11. A mind is not a "machine". A machine produces the outputs as designed by the mind and therefore presupposes the mind. A calculator outputs 4 for input 2+2 because a mind told it to do so.

      Delete
    12. Apologies Don, since you began talking about what other people and philosophers would say about qualia, I took you to be discussing qualia as viewed by many other philosophers, not your own personal view of materialism.

      Further, you need now to justify some things, since you claim now to have a view of materialism. When you originally stated it was your assumption that material mind can sense qualities of matter, I took it to be such. But now I understand that you can justify this claim as congruent with your view of materialism. Is it that colours are an actual part of matter?

      Delete
    13. Anonymous,

      "It doesn't make sense for us to talk about the 'remembering' of the song without also talking about the experience of *what it is like* to conjure that memory into the forefront of our awareness and experience it there."

      This is an interesting issue (and I'll quibble with your verb tense). How far is remembering *what it was like* from experiencing "what is"?

      Beethoven could 'hear' as he created his 9th Symphony even though he was deaf. He could remember quite well how instruments used to sound. But I've always felt kind of sad that Beethoven never actually heard his 9th.

      Most of us don't playback music in our heads to the degree that we don't want to hear the CD again and don't want to hear it live in concert.

      I'm skeptical that mere words are good teachers. This is particularly true with human things (rather than intellectual exercises like math). Most people put a lot of faith in words. A great storyteller can conjure up all sorts of emotions in others. But how effective is this as a learning tool, really?

      I've come to the conclusion that nothing substitutes for experience. That romance on screen, where a man brings a woman to tears (and maybe us) then apologizes and all ends well -- that is a fleeting lesson. It doesn't stick like the thing happening to us in life. That lesson in real life doesn't always stick either, but, imo, it's the only thing that can stick.

      You can tell a child the oven is hot, so don't touch. The child knows what 'hot' means, kind of. But only when he burns himself does he really stay clear of it. It takes the real-time qualia to burn that memory into us.

      I can remember 'Ode to Joy' quite well. But in no case does my remembering stir my emotions like hearing the thing through my ears. Remembering can't reproduce in me the *what it is like* to hear it live. It's not even close.

      In my life, remembering is a shadow of experience. So I'm skeptical that remembering qualifies as qualia. Do philosophers talk about degrees of qualia? I don't know. Maybe this issue should prompt that train of thought.

      But maybe I didn't address your idea that seems to be that merely remembering is a "what it's like* of its own -- what it's like to think about anything. That could take much ink.


      "I'm still unsure how this really helps to answer the HP, which deals with the existence of qualia as I defined the term."

      The short answer is that I see qualia as tangent to the real problem which, imo, is consciousness. If we answer that, maybe the rest will fall into place. The longer answer would delve into my distaste for using subjective, 'felt' experience as a window into ultimate truths.


      "if our hands are forced we should always prefer to adopt [brute fact] "sooner" rather than 'later' in our ontology."

      I agree. This is one of my main problems with the philosophy promoted here. I believe they are bringing in brute fact way too early. Along this line, you say it's taking too long for a materialist explanation for consciousness. But I think technology is required to solve this problem and the technology is not quite powerful enough yet. So I would expect more theory than practical progress, and I'm not confident theory itself will convince many -- certainly not me.

      "consider how physicists have adopted certain things into their ontology when necessary."

      Physicists can be wrong. I'll go further. Physicists *are* wrong -- they just don't know what they're wrong about. Bottom line for me is this: Is a theory empirically falsifiable? If it isn't, I see no value in it.



      Delete
    14. Anonymous - Tom Cohoe,

      "You are besotted with computation as comprehensive of reality when computation is just an image of reality."

      I'm very familiar with the fact that computational models are not reality.


      Delete
    15. DrYogami,

      "In your analogy a microcontroller 'queries sensors and processes data', yet with the caveat that the micro does not have qualia. If that's the case, why is any qualia necessary at all? Are you claiming that it's a 'spandrel', a byproduct of something that has utility? Or that it has utility in itself?"

      I'll make a guess.

      Our bodies have sensors that don't provide us with qualia -- parts of digestion, circulation, etc. Those systems don't need our conscious choice. Our choice might throw havoc into the whole thing. Plants sense and react to the environment. But as far as we know they have no qualia because they lack consciousness.

      We could be zombies and react unconsciously to all sensory input. But reaction alone would not allow us to adapt to a fast changing environment. I believe consciousness is a tool to help us adapt. We react instinctively to fire on our fingers. So if we didn't need our fingers for anything other than pain avoidance, I suppose there would be little need for qualia of touch. But we use our fingers for way more than that -- fine detailed stuff. It pays off to let touch rise to the level of qualia so we can consciously analyze and manipulate things. As you suggest, qualia could be an evolutionary byproduct that started with dumb sensory reaction but morphed into something much more useful.



      Delete
    16. Journey 516,

      "Is it that colours are an actual part of matter?"

      Color is how we sense a specific range of light reflected off matter. The light is not part of the surface we call red, but the reflective properties of that surface are part of that surface. Color itself is derived from our human sensory system. The claim that "qualia of our experience of the apple [does not] correspond to anything in the apple itself" strikes me as a straw man. Maybe someone believes this, but not me. Of course there are properties in the apple that correspond to our experience of the apple. It really does reflect light of certain wavelengths. But is our experience of the apple the totality of the properties of the apple or the best description of the apple? No.

      Delete
    17. What color is a proton? It must have a color, right? Or, if you prefer, "reflectence property", right?

      What color is a quark? Or whatever the smallest particle is. What color is it?

      Delete
    18. @ Don Jindra,

      "As you suggest, qualia could be an evolutionary byproduct that started with dumb sensory reaction but morphed into something much more useful."

      Qualia (a word that I avoid if at all possible) evolved? From what? I have to say that you are not making any, uh, ... sense.

      Tom Cohoe

      Delete
    19. Hi Don, First Anonymous here:

      1.) A very enjoyable introduction you give here. As you surmise towards the end, my main point was that remembering an experience one had (i.e."playing it back"), though not the same as the experience itself, is an instance of conscious experience in its own right and therefore counts as qualia on the definition I'm using. At any rate, even if we're excluding the kinds of qualia you're talking about, we're still left with other kinds of conscious experience. And the HP is about how/why *any* kind of conscious experience exists -- so I'd say we haven't made any real progress.

      2.) Re: brute and basic facts , I actually think we disagree. By "sooner" and "later" I mean in terms of logical priority, not in temporal sucession. You seem to be criticizing people like Feser, other commenters, and myself for wanting to take some aspect(s) of consciousness as an ontological primitive (not in the substance dualism sense, mind you), before relevant technologies have developed sufficiently to conduct an exhaustive empirical investigation of the issue. So in that sense you personally *don't* want to take something as basic "sooner" rather than "later."

      By contrast, I meant to criticize the kind of materialism I often see that doesn't take any aspect(s) of consciousness as a primitive in its ontology, yet far down the metaphysical line is content to take as a brute fact that human consciousness exists at all. "Yes, philosophical zombies are logically possible, demonstrating there's no actual causal/explanatory connection between the physical brain and consciousness. But in our universe there just so happens to be a law that a physical system like our brains is conscious." That's like if halfway through a math textbook, when the author realizes they are unable to demonstrate a certain result, they said: "We take it as a given that this result follows from what we've shown." "No no no," I'd object, "it doesn't work like that -- that's how you get inadvertant contradictions. You need to adopt the simplest axioms you need at the outset and then show how the result follows from there." You can't have a gaping explanatory hole in the middle of your system. And papering over it with a "law" wouldn't help. Ignoring how this would imply a controversial account of what natural laws even are, it would be immensely strange to think a specific natural law would exist as a brute fact for something as particular, contingent, and cosmically isolated as human consciousness. In the end, I insist that the physicalist argue how consciousness can be accounted for in purely physical terms -- anything short of that and we must revise our "axioms." That's what I meant by "'sooner' rather than 'later.'"

      I'm not accusing you of taking this kind of approach, by the way, it's just what I was referring to.

      (see next comment for part 3):

      Delete
    20. Don,

      3.) Re: theory vs. empirical observation, as a preliminary I'm going to flag that I'm not too informed on philosophy of science.

      3a.) I'd say some kind of theory is needed to guide empirical observation from the outset. Otherwise we'd have no idea where or how to look for data in the first place. But the theory itself could change very quickly if it ends up conflicting with what we find. Where I see materialist scientists and philosophers is having no useful framework to even begin investigating how matter can produce mind. Maybe they'll arrive at it by groping around in the dark experimentally, but I'm doubtful.

      3b.) I think neuroscience has developed to the point where we can imagine what having an exhaustive physical model of brain function would be like. It'd be an extension of the kinds of knowledge we already have. Imagine that we were able to map a brain in its entirety down to the subatomic level. We'd be able to describe and predict every brain function in entirely third-person terms. But then I'd just be more confused! If first-person experience need not be invoked to explain anything, I'd be deeply puzzled as to what it's doing there in the first place. On the other hand, if it's not causally invovled, maybe we'd see how what we call "first-person experience" is a necessary byproduct of the physical brain. Note the inclusion of "necessary" to avoid the "it just does" attitude I addressed in 2.)

      This would amount to an empirical demonstration of the fact that p-zombies are logically impossible given the physical laws and entities in our universe. But I think we have excellent arguments already to suppose that they are indeed logically possible. That's beyond the scope of our discussion here, but I hope that clarifies why I don't have faith that neuroscience will explain consciousness on a purely physicalist ontology.

      4.) Addendum: This was not raised in our exchange, but elsewhere in this thread you propose why consciousness is evolutionarily useful. I always find these arguments from physicalists deeply, deeply puzzling. After all, it is precisely on physicalism that we can describe and predict the behavior of an organism in purely third-person terms.

      "But we use our fingers for way more than that -- fine detailed stuff. It pays off to let touch rise to the level of qualia so we can consciously analyze and manipulate things."

      The irony in a sentence like this is that now *you're* the one hiding behind the kind of vague subjective language you said you detested! On physicalism, if I were an omniscient neuroscientist I could give perfectly precise third-person descriptions of what you're saying in purely physical terms:

      "'Rise to the level of qualia,' 'consciously analyze,' -- what vague claptrap is this?? It's purely these neurons firing in this and that way. See Figure #4 for all the details in my paper."

      The upshot of this is that any reference to consciousness as being "evolutionary useful" falls flat, insofar as we can refer to and imagine the physical system producing these beneficial adaptations without any subjective experience at all. It's clear now how this loops around back to p-zombies and other related arguments. I'm not trying to rehash those here, I'm only trying to "defend theory" by showing how all of these discussions must inevitably loop back around to the philosophical arguments -- regardless of how much empirical knowledge we have.

      Delete
    21. Don Jindra,

      The scenario you're drawing only makes sense if qualia/consciousness aren't epiphenomenal, but actually have causal powers in their own right. Do you subscribe to physical causal closure?

      Delete
    22. Alex,

      "At any rate [...] we're still left with other kinds of conscious experience. And the HP is about how/why *any* kind of conscious experience exists -- so I'd say we haven't made any real progress."

      Progress, maybe not.

      But can we think about things for which we have no experience? Even if we let our minds create a futuristic utopia, that new world is based entirely on reconstructed pieces of past experience.

      Starting there, I'm going to brainstorm.

      Thinking itself is the process of replaying our past experiences in an effort to build a kind of jigsaw puzzle where a wide variety of old pieces come together into a more pleasing or useful whole. Rearranging the furniture -- that's what thinking is. There can be no thinking (processing) without experience (qualia). But I still see them as functionally separate.

      The mere process of rearranging mental furniture might be contemplated as a qualia too, but is that concept of practical value? We might as well stop asking, "What is it like to be a bat?" The bat's experience of being a bat boils down to "What is it like to think about experience?" Whether you're a bat or a human, it's the same question with the same answer: Qualia is all in the mind. The body has been rendered an extravagant input device (my position anyway).

      If thinking is only that process of rearranging furniture (past experience) into a more pleasant whole, I see no reason to add a new 'substance' into the mix, a 'substance' that by definition can't be experienced and can't be furniture. The 'substance' would have to be the moving itself -- a verb instead of a noun, a kind of fundamental force that appears only in the mind.

      This wouldn't be too far off from what I think about consciousness. But in my thinking, consciousness and will (both seen as as verbs) can develop out of complex processing systems like our brains, and (eventually) be fully explained by that processing.

      Delete
    23. Alex,

      "By 'sooner' and 'later' I mean in terms of logical priority, not in temporal succession."

      Or both.

      I don't have confidence in the truth-finding merits of purely logical arguments. So when I see a logical proof that has, at best, precarious attachment to empirical roots, I'm skeptical. The first assumptions of such a proof often strikes me as a sort of 'brute fact'. Whether we start as a monist or dualist, we start with what we consider a brute fact. The question for me is, is only logic permitted from that point on?


      "Yes, philosophical zombies are logically possible, demonstrating there's no actual causal/explanatory connection between the physical brain and consciousness. But in our universe there just so happens to be a law that a physical system like our brains is conscious."

      I have several problems with that assertion. So we agree there.


      "it would be immensely strange to think a specific natural law would exist as a brute fact for something as particular, contingent, and cosmically isolated as human consciousness."

      I agree it would be strange. I do not see consciousness as a brute fact. It must be explained in full. But I'll insist that to account for it with a dualist 'substance' is to account for it as a brute fact.

      On p-zombies:

      If thought experiments don't lead to something that's empirically testable, they're wild speculation or hide dubious assumptions.

      If a p-zombie is logically possible, it does not follow that there does not exist a better way for biology to solve the problem. It's logically possible that birds could be fixed wing beings and grow propellers. But that's an inefficient way to build a flying animal. The hidden assumption in the p-zombie analogy is that consciousness is not a vast improvement in processing efficiency.


      "The irony in a sentence like this is that now *you're* the one hiding behind the kind of vague subjective language you said you detested!"

      Consciousness is subjective from our personal POV, but existence of consciousness is not subjective. We are certainly conscious animals. So are many animals, apparently. So I agree there should be an evolutionary benefit for it, or it shouldn't exist.


      "'Rise to the level of qualia,' 'consciously analyze,' -- what vague claptrap is this?? It's purely these neurons firing in this and that way. See Figure #4 for all the details in my paper."

      And I'd answer, Mr. Neuroscientist, figure #4 does not have enough pages to explain the complex algorithms and data structures I'm talking about.

      I'll elaborate. There are collections of neurons that could be 'programmed' to do complex tasks. Computers have already proven the possibility. We're going to have self-driving cars that perform pretty well. But how long did it take us to learn how to drive? A couple of weeks of an hour per day? After that, we were capable of handling virtually any circumstance. And this is but one minor task our brains can master, and master quickly. How big will that p-zombie brain have to be to handle the infinite variety of tasks we confront in the world? How many millions of years will it take to evolve that zombie brain that can handle the situations we handle easily every day? Those are the questions evolution has to answer. It could very well be that our consciousness in a necessary condition to get maximum efficiency out of minimal brain size. We do not know yet. So it's going to be a while before we can say with confidence that the p-zombie is a practical competitor to our adaptability. Its head may have to be a million time bigger than ours. That p-zombie may be a couch potato by physical necessity.

      Delete
    24. Anonymous - Tom Cohoe,

      "Qualia (a word that I avoid if at all possible) evolved? From what? I have to say that you are not making any, uh, ... sense."

      Yes, our ability to be conscious of experience evolved, like our digestive system evolved. You may not believe in evolution. I wouldn't know what you believe. But in the 21st century surely it makes sense when someone suggests this could be the case.


      T N,

      Why would a proton have color? Things don't need a color. Glass usually has no color. Something as black as coal has no color. Something has to reflect light our eyes can detect to have what we call a color. As far as I know, protons cannot reflect light because they're too small.


      DrYogami,

      Since I do not believe there is anything but the material, I must subscribe to physical causal closure. Mental events are caused by physical events, and in fact are physical events. This is true even when daydreaming. But I do not believe thinking about killing my neighbor's dog necessarily results in me killing it.

      Delete
    25. @ Don Jindra,


      "Yes, our ability to be conscious of experience evolved, like our digestive system evolved."


      I didn't question that it evolved. I asked "from what?". Evolution is change in something that already exists. So, again, from what?


      "You may not believe in evolution. I wouldn't know what you believe."


      "Believe in" is a funny way to put it, but I have no problem with the theory of evolution. Michael Behe and Richard Dawkins both turn my stomach.


      "But in the 21st century surely it makes sense when someone suggests this could be the case."


      You haven't made sense to me yet. You told me that you were well aware "that computational models are not reality", but you propose a computational model of sensation:


      "As a firmware engineer, this is the way I look at it. A microcontroller can have various sensors attached: maybe an accelerometer, a magnetometer, a thermometer. A protocol allows the microcontroller to query the sensors, then the microcontroller processes that data. These are distinct things: sensing, communicating, processing. Qualia for me is equivalent to the communication protocol. Qualia is like an "agreement" between microcontroller and sensor in how they exchange data (this is analogy since the micro does not have qualia). Qualia is a means to an end, not the end. Qualia (communication) is a functionally necessary part of how the conscious machine receives data, but not in how it processes the data. So I do think there is an important distinction."


      This description of "reality" as you see it is not modeled in computation? But, "how it processes the data"? Again, you are computation besotted.


      Furthermore, since you reject the idea of non-material effect in this world (what then would be the basis of selection in a world ruled by the uncertainty principle?) you are not saying anything that could distinguish between non-materialism and materialism. If you assume materialism is the way it is, you, according to what you have said, have to demonstrate materialism experimentally or according to you it's just empty talk.

      Tom Cohoe

      Delete
    26. Don Jindra,

      Okay, so you don't believe that there's anything other than matter. Mental events are caused by physical events--and even are physical events. Could you describe what you mean by 'material' or 'physical'? Because if everything is 'material', I'm trying to understand what the use of the concept of 'matter' is and what differentiates it from anything else.

      Delete
    27. Don Jindra,

      You say above that it is a straw man to claim that anyone denies that qualia corresponds to something in the substance itself. Then you go on to deny that qualia corresponds to something in the substance itself: you say that protons don't need a color. (Or, if you prefer, they do not possess the properties that correspond to qualia in an observer)

      As we collect protons together into a larger substance, where does the color come from?

      Perhaps properties inhere to substances apart from their constituent parts? Are you sure you disagree with the philosophy expressed by this blog? You're not much of a materialist.

      Delete
    28. Tom Cohoe,

      "Evolution is change in something that already exists. So, again, from what?"

      I found an interesting paper on the topic.

      https://www.pnas.org/content/113/18/4900

      Sure, there's a lot of speculation. But it does seem plausible to me that the evolution of consciousness went something like this: Animal mobility led to a need to sense the environment which led to the organism's need to be 'aware' of its place in the environment which led to a need to distinguish itself from its environment ("I must not eat my leg!") which led to a sense of a subjective experience of its environment which led to conscious placement in its environment.


      "You told me that you were well aware 'that computational models are not reality', but you propose a computational model of sensation..."

      All thought, all communication is based on models. The words we share right now are symbols constructing models. There is no way around this. I used a computer analogy as a model. I tried to make it clear I used it as an analogy. If you don't think the analogy fits, then it would be helpful if you offered one you think does fit, or explained why mine doesn't.


      "(what then would be the basis of selection in a world ruled by the uncertainty principle?)

      I don't understand the question. The uncertainty principle does not imply we can't select between two options about reality. It does not imply every option is equally plausible. It does not imply total chaos. It states we cannot know some things.


      "since you reject the idea of non-material effect in this world ... you are not saying anything that could distinguish between non-materialism and materialism."

      I don't want to stray too far from the issue I first commented on, which was focused on qualia and consciousness. But if this "non-material stuff" affects the material, what makes it immaterial? OTOH, if this "non-material stuff" cannot be detected by any known physical means, why should I believe it's really there? I have very low confidence in anything that cannot be detected through instruments.


      "If you assume materialism is the way it is, you... have to demonstrate materialism experimentally or according to you it's just empty talk."

      Sticking to the qualia issue, I agree. When we create consciousness in our machines we will have an answer gained through much experimentation. OTOH, the dualist will never be able to experimentally prove his position. This probably doesn't disappoint one who has high confidence in logical proofs. But I have little confidence in such proofs.

      Delete
    29. T N,

      "Then you go on to deny that qualia corresponds to something in the substance itself: you say that protons don't need a color."

      There is no contradiction. If we perceive no color in an object, there is probably nothing to correspond to. A red stop sign is red. It's not green. So there is nothing in the substance of the stop sign itself that *can* correspond to green (ignoring the white letters). If an object has no color, there is nothing in the object for a color to correspond to. One could say the object was colorless, and then I would agree colorlessness corresponds to the object itself.

      To clarify, I said this sentence was a straw man: "Hence the redness, sweetness, fragrance, and crunchy sound of the apple are taken to be nothing more than the qualia of our experience of the apple, and not to correspond to anything in the apple itself. "

      "Correspond" is the key word. "Red" corresponds to the composition of the apple in that the composition reflects and absorbs different wavelengths of light. That composition is in the apple. "Red" itself is not *in* the apple, it's how we perceive it through our eyes. The straw man is the implication that a materialist must claim "red" is a disjointed property that just happens to arrive in our mind when we see an apple. I know of no biologist who would say the apple has no property that corresponds to the way we see the apple. The apple has something in itself that we do sense. But what we sense is a filtered, inexact and indirect version of what is there.

      It's tempting to ignore this straw man as clumsy wording. But it's exploited. We're led to a dystopian materialist reality that is a "desiccated mathematical husk." So I cannot leave it as clumsiness. There are people with all sorts of beliefs who look at reality through cynical eyes. Materialists are not led there by necessity. I doubt materialists are more prone to cynicism than any other group. One can try to look at reality objectively and still subjectively enjoy it.


      "Perhaps properties inhere to substances apart from their constituent parts?"

      No. The arrangement of constituent parts forms its properties, like the constituent parts of an internal combustion engine create properties for the engine that mere clumps of metal do not exhibit. These new properties are entirely physical.

      Delete
    30. DrYogami,

      I define the 'material' or 'physical' as those 'things' that can be detected by ourselves or our instruments (and by instruments I do not include crystal balls).

      Delete
    31. This comment has been removed by the author.

      Delete
    32. So for something to count as 'material' it has to be detectable by ourselves or our instruments. This would seem to rule out the mind. I mean, that's the thrust of what's been argued over and over: science deals with the objective third person features of the world.

      What you are proposing in your hypothetical scenarios for how consciousness could 'evolve' somehow from a physical system basically amounts to: magic. Poof! Qualia appears. Didn't know you believed in miracles.

      Delete
    33. “The apple has something in itself that we do sense.”

      Great! You agree with A/T.

      “I know of no biologist who would say the apple has no property that corresponds to the way we see the apple.”

      I’m not sure what biologists have to do with it, but OK, you don’t know any biologists who are eliminative materialists. OK.

      You deny my statement that substances have properties apart from their constituent parts, and then you give an example (an engine) of that very thing: engines have properties that clumps of metal do not. Water has properties that hydrogen and oxygen do not. Etcetera.

      OK, I’m tired of sorting through this. So to your mind, you can just poo-poo the entire high-level debate on this issue by some of the best atheist and theist minds alike, because to you these people are just too silly to see the obvious. OK.

      Delete
    34. @ Don Jindra,


      "the dualist will never be able to experimentally prove his position"


      Don't call me a 'dualist' unless you are referring to our conversation as a 'dual' (one which grows more friendly then more angry). :-) 


      The concept of dualism is hopelessly muddled. I believe in the concept of a god that has no parts, that is the simplest existing thing. Is that dualism? OTOH, Aquinas held that every material thing had form and matter. That's a dualism, but it has nothing in common with the dualism of most modern philosophers. Call me a 'hylomorphist'



      "But if this 'non-material stuff' affects the material, what makes it immaterial?"


      Randomness in position and momentum (material properties of state), consequent upon Bohr's principle of complementarity from the uncertainty principle, means that the state of matter is only statistically meaningful. Within a statistically well defined distribution the components of the state of _individual_ particles are randomly distributed. Note the distinction of _statistical_ distribution and individual distribution. EG, a sample of radioactive material displays a well known statistical distribution of times between individual decays, but the time to the next decay is random within that distribution, and more tellingly, the next particle to decay is random.


      To cut this short, tiny differences in the group state of a large number of particles (EG, the atmosphere) grow into unpredictable in principle weather systems. We can predict that the hurricane will hit Louisiana when it's a day out, but we cannot predict which mini-cell in the gulf will grow into the hurricane, or whether any will. Ever seen a dust devil?


      Is randomness a material? No? Does it affect the material world? You bet. It affects its distribution. There's the answer to your question.


      Now you have to explain how this immaterial randomness doesn't exist or prove it experimentally. You can't do it and your materialism is nothing but failed circularity.


      An image of God is an infinite, unbiased, bit-by-bit-random binary sequence. It must contain every possible subsequence, in the density required for the subsequences' distribution to not be a bias in the infinite sequence. Only an infinite sequence can meet this statistical property so a finite sequence can, at best, be 'disordered'.


      I can guarantee that it is impossible to have a comprehensive understanding of the infinite sequence, yet this is a limited image of God. I can make a good case that mutations in the total genomic content of the Earth is _random_, although it is probably biased, but that's not a problem. It is a finite sequence. The randomness (rather than disorder) comes from the algorithm which produces them in time. The finite sequence of mutations is a subsequence of the God image.


      Selection for survival also has randomness built in.


      But that's enough.


      Tom Cohoe

      Delete
    35. @Don Jindra:
      "Sure, there's a lot of speculation. But it does seem plausible to me that the evolution of consciousness went something like this: Animal mobility led to a need to sense the environment which led to the organism's need to be 'aware' of its place in the environment which led to a need to distinguish itself from its environment ("I must not eat my leg!") which led to a sense of a subjective experience of its environment which led to conscious placement in its environment.

      And this is falsifiable how? Because according to you, what it's not "empirically falsifiable" is useless.

      Delete
    36. T N,

      You began with this: "What color is a proton? It must have a color, right?'

      You continued with this: "Perhaps properties inhere to substances apart from their constituent parts? Are you sure you disagree with the philosophy expressed by this blog?"

      I think it was reasonable for me to believe that on some level you think properties inhere to substances apart from their constituent parts. I also believe *some* properties inhere to substances apart from their constituent parts, as in your example of water. But secondary properties do not inhere to substances, though substances do have properties that give us the sensation of those secondary properties. So there is correspondence between properties in the substances and our perception of them. So I differ with the philosophy expressed in this blog. I'm wondering why you insisted protons have to have a color. It seems to have led nowhere even though you struck me as adamant about the necessity of it. So honestly, I don't know what your complete position is.

      Delete
    37. DrYogami,

      "So for something to count as 'material' it has to be detectable by ourselves or our instruments. This would seem to rule out the mind."

      You're communicating with me. To me that counts as detection of a mind. We can also detect the brain's activity. The technology of detecting the mind will get better over time. So I disagree the mind is not detectible.


      "What you are proposing in your hypothetical scenarios for how consciousness could 'evolve' somehow from a physical system basically amounts to: magic. Poof! Qualia appears. Didn't know you believed in miracles."

      Do you believe our arms evolved through magic too? Our eyes? If not, why do you single out our minds?

      Delete
    38. Tom Cohoe,

      If I describe a child as a brat, brattiness doesn't force the child to behave like a brat. She behaves that way. The description doesn't make brattiness a causal substance. It may be an effect from something else (bad parenting) or it may be the way the child is (unruly material), or it may be I expect too much from children.

      The randomness you describe is a behavior of matter. Randomness doesn't move matter -- it's not a force. It's a descriptive snapshot of how it may move. Descriptions are for humans.

      We could say the same of order.

      Delete
    39. UncommonDescent,

      I described what I think is a plausible scenario. I was not insisting this is the truth of the matter. I'm sure some clever neurologist could setup an experiment to attempt to falsify pieces of that scenario. But maybe your objection is that evolution itself is not falsifiable?

      Delete
    40. @Don Jindra:
      I described what I think is a plausible scenario. 

      You said that "unfalsifiable speculation" is "useless". So what's the point of using speculation to support your theory of "how consciousness evolved"? Your scenario is just a nice just-so-story with zero empirical support (both present and future). Just-so stories may sound "plausible" but without proof, they amount to nothing.

      I'm sure some clever neurologist could setup an experiment to attempt to falsify pieces of that scenario.

      I'm going to baptize this as the appeal to the lab-coat fallacy ;) Unless you can explain us how the suppossed evolution of consciousness is falsifiable, what you wrote is just an empty appeal to some nebulous future where some nebulous "neuroscientist" will solve the problem that is burning your hands.

      Meanwhile, the "evolution of consciousness" is just an assertion without proof and therefore it has zero validity.

      Delete
    41. @ Don Jindra,


      "Randomness […] is a behavior" (of something, which something must be material according to you) but the behavior doesn't affect material?


      Behavior affects distribution. It makes it unpredictable on all scales. Yet when we measure the distribution, we find a particular one. The hurricane did hit Louisiana. So what causes the particular distribution found out of the many possible? Have you given up on cause?


      Neurons also fire spontaneously at a low rate, an individually unpredictable, random, behavior.


      Tom Cohoe

      Delete
    42. Don Jindra,

      You're communicating with me. To me that counts as detection of a mind. We can also detect the brain's activity. The technology of detecting the mind will get better over time. So I disagree the mind is not detectible.

      If the fact that I'm communicating with you counts as detection of a mind and therefore material, and the fact that we can detect brain activity also is what makes it 'material', then 'material' or 'physical' is a really misleading term for what you're describing. 'Idealism' would be better.

      Do you believe our arms evolved through magic too? Our eyes? If not, why do you single out our minds?

      Well, because the cases just aren't on a par. As even Sam Harris wrote (in The End of Faith no less):

      "The idea that brains produce consciousness is little more than an article of faith among scientists at present, and there are many reasons to believe that the methods of science will be insufficient to either prove or disprove it."

      Delete
    43. @Don

      "I define the 'material' or 'physical' as those 'things' that can be detected by ourselves or our instruments"

      Well, our and our instruments can only really detect qualities like size, color, shape etc, but we can't detect something "behind" these that unites and organizes the qualities on one thing. How would you respond to a Hume or a Mill, that would say that you have no grounds to believe in the existence of substances, material or whatever, behind qualities?

      Delete
    44. Don,

      I don't think I saved my previous reply...in any case, while I have thoughts on everything you've said I'm going to limit my response to your comments on p-zombies and the evolutionary utility of consciousness:

      On physicalism, the function and therefore efficiency of the brain is entirely determined by its physical structure and processes. A p-zombie is defined as a creature 100% physically identical with a human but lacking any inner subjectivity/conscious experience. So it's incoherent given our definitions to suppose that a conscious brain might be more efficient than a p-zombie one and thus try to explain the evolution of consciousness that way. Alternatively, if you think a physical brain like ours must be conscious, then you're back to denying the logical possibility of p-zombies.

      The point of the thought experiment isn't to consider a hypothetical creature that may have evolved alongside or instead of us -- it's just to tease out how merely considering the physical facts of the brain doesn't allow us to infer the existence of consciousness (https://en.wikipedia.org/wiki/Further_facts). Many objections to the p-zombie experiment I find are based on this sort of misconception (e.g. the charge that it implies solipsism).

      Delete
    45. UncommonDescent,

      I wrote, "Is a theory empirically falsifiable? If it isn't, I see no value in it." I did not propose a *theory* on the evolution of consciousness. I specifically said I was merely proposing a plausible scenario (that is, I speculated it could have gone "something like this."). I chose my words carefully so as to not be proposing a theory. It was you, UncommonDescent, who wrote: "And this is falsifiable how? Because according to you, what it's not 'empirically falsifiable' is useless." If you want to use my words against me, please try to use my actual words. You have broadened the scope of what I said. I will not defend a stranger's interpretation of my words.

      "Meanwhile, the 'evolution of consciousness' is just an assertion without proof and therefore it has zero validity."

      You seem to think I'm a logical positivist. But I'm not. So I'm not obliged to respond to that. Even if my speculation has zero validity, exactly the same can be said for those who 'explain' consciousness via empty appeals to some nebulous 'substance'.

      Delete
    46. Tom Cohoe,

      "'Randomness […] is a behavior' (of something, which something must be material according to you) but the behavior doesn't affect material?"

      The description -- the characterization -- does not affect material. Randomness is a characterization. It's like me saying, 'Audrey Hepburn moves elegantly.' My description of her behavior doesn't make her behave in that manner. My conception of elegance itself does not make her behave in that manner. Likewise our characterization of randomness does not make matter move randomly. There is no 'random' force that we know of. OTOH maybe we ought to look, if that's even possible.

      I have not given up on cause (although we don't know what cause is). I have given up on 100% determinism. The double-slit experiment seems to show this is not a deterministic universe.

      Delete
    47. DrYogami,

      "If the fact that I'm communicating with you counts as detection of a mind and therefore material, and the fact that we can detect brain activity also is what makes it 'material', then 'material' or 'physical' is a really misleading term for what you're describing. 'Idealism' would be better."

      I believe there is a reality that is totally free from how our minds interpret it. It exists the same with or without us.

      I finally did read Sam Harris's 'The End of Faith.' I disagree with most of what he wrote. I think he's a fraud. I believe his attack on free will is no more than an article of faith. I don't even like many of his attacks on religion. Then near the end he started pushing a religion of his own. The guy is a hypocrite.

      Delete
    48. Don Jindra,

      I believe there is a reality that is totally free from how our minds interpret it. It exists the same with or without us.

      Well, that's why I interpreted your earlier response as implicitly Kantian. Your definition of 'material' involves the observer as part of the theory. You're getting into the noumenon.

      I finally did read Sam Harris's 'The End of Faith.' I disagree with most of what he wrote. I think he's a fraud. I believe his attack on free will is no more than an article of faith. I don't even like many of his attacks on religion. Then near the end he started pushing a religion of his own. The guy is a hypocrite.

      Well, yeah. They all do this to some extent. Even in the bibliography to Richard Dawkins' 'The God Delusion', he includes a book by an author named Ursula Goodenough titled 'The Sacred Depths of Nature'.

      Delete
    49. @ Don Jindra


      Between two successive nuclear decays in a sample being watched, there is a time, delta_t. What caused this time interval to be delta_t rather than delta_t/2 or 8*delta_t? If you accept indeterminism, as you say you do, then what caused the particular result? Nothing? Nope, from indeterminate possibility came a determinate result. What caused the change?


      Neurons also fire in a random way, probably some amplified quantum mechanical indeterminacy. A quantum mechanical amplitude (https://en.m.wikipedia.org/wiki/Probability_amplitude) for the random firing of a single neuron means that there is also an amplitude for the random firing of any combination of neurons in any time sequence in the entire ensemble that constitutes the brain.


      How are you going to incorporate this non Turing brain activity into your Turing robot?


      Tom Cohoe

      Delete
    50. @Don Jindra:
      I wrote, "Is a theory empirically falsifiable? If it isn't, I see no value in it." I did not propose a *theory* on the evolution of consciousness. I specifically said I was merely proposing a plausible scenario (that is, I speculated it could have gone "something like this.").

      My bad :) Now let's cut to the chase because all this word-twisting is kinda tiresome.

      What is exactly what we are discussing here? Because if it's only a question of beliefs , then your beliefs are yours, and my beliefs are mine, and Prof. Feser's beliefs are his and then we can continue with this ad infinitum...

      If what we are discussing is a matter of objective truth, then I want you and your side (the side that claims that "consciousness has evolved and it's the *creation* of matter") to show us *proof* so we will shut our mouths and stop saying that *mind* is the ontological primitive in this Universe (the mind of God of course and everything stems from there).

      Even if my speculation has zero validity,

      It has zero validity.

      exactly the same can be said for those who 'explain' consciousness via empty appeals to some nebulous 'substance'.

      Well, if the side that appeals to what you describe as a 'nebulous substance' does not violate the laws of logic, then that side is obviously superior to yours (the physicalist). Because your side has yet to answer the most pressing question of all: can a physicalist claim ownership of his/her thought processes:
      a) yes?
      b) no?

      Because if he/she can not claim ownership of his/her thought processes, he/she is reduced to the level of a string puppet that should not be taken seriously at all.

      If we are physical creatures subject to the Laws of Physics, then our thougths are subject to those same Laws of Physics and they are not *ours* and then we are causally impotent creatures devoid of free will and nothing makes sense anymore.

      Delete
    51. Talmid,

      You expect to unite and organize size, color, shape etc. but I don't know what you mean.

      "How would you respond to a Hume or a Mill, that would say that you have no grounds to believe in the existence of substances, material or whatever, behind qualities?"

      As far as I know, both Mill and Hume were empiricists who believed knowledge is gained only through experience of the natural world. You imply they also believed there may be nothing of substance driving that experience. What text are you referring to?

      Delete
    52. Alex,

      "A p-zombie is defined as a creature 100% physically identical with a human but lacking any inner subjectivity/conscious experience."

      I'm saying that an actual creature physically identical with a human but lacking any inner subjectivity/conscious experience is a huge leap of faith.

      This is how we got here:

      You wrote: "The upshot of this is that any reference to consciousness as being 'evolutionary useful' falls flat, insofar as we can refer to and imagine the physical system producing these beneficial adaptations without any subjective experience at all."

      I absolutely deny this. One cannot posit a creature that is theoretically robbed of what may be an evolutionary useful trait and then use that fiction to say one thing about the trait's actual usefulness in the evolution of the real creature. That's playing nature's god. That's assuming the conclusion.

      I'll elaborate.

      There is no reason to think a p-zombie could exist in an equivalent human package. It's philosopher's hubris to claim he knows as much as nature, that nature could not possibly have come up with a brilliantly economical solution, a solution far better than the philosopher could ever imagine. IOW, the philosopher assumes nature had no 'technical' reason to build a conscious brain, then the philosopher eliminates the 'technical' reason and pretends the behavioral result is the same. He rigs it so he can conclude consciousness has no technical or logical reason to be there, that it's excess baggage. The p-zombie definition is a hidden form of begging the question.

      I deny the technical practicality of p-zombies. Consciousness is a technical problem. It's not a logical problem. A mere definition cannot change the problem from technical to logical, or from nature to storytelling.


      Delete
    53. DrYogami,

      "Your definition of 'material' involves the observer as part of the theory."

      Since I believe 'the material' is the same with or without us, I don't think the observer is relevant. I'm interested in the epistemology. We can't possibly know about something we or our instruments cannot sense. If we can't know something exists, I don't see how speculating about it can add to our knowledge base.

      Delete
    54. Tom Cohoe,

      "How are you going to incorporate this non Turing brain activity into your Turing robot?"

      A brain is not a Turing machine.

      I don't understand your question about successive nuclear decays. The delta_t time is indeterminate. It looks like you are asking me to explain why something is indeterminate.

      OTOH maybe you're asking how indeterminate individual states of decay could lead to a highly predictable decay of the whole? like Gaussian distribution? the implication being that the small cases of randomness average out to irrelevancy? If this is the case, my answer is that even a small bit of indeterminacy in the universe destroys the extreme position that everything in the universe is determinate. It swings the door wide open to other unknowns: "There are more things in heaven and Earth, Horatio than are dreamt of in your philosophy."

      Delete
    55. UncommonDescent,

      "If what we are discussing is a matter of objective truth, then I want you and your side (the side that claims that "consciousness has evolved and it's the *creation* of matter") to show us *proof* so we will shut our mouths and stop saying that *mind* is the ontological primitive in this Universe (the mind of God of course and everything stems from there)."

      We basically agree here. This issue is never going to be settled by words. It's incumbent on my side to demonstrate consciousness can exist in a machine we create. Time will tell.

      Can a physicalist claim ownership of his/her thought processes? I can and do claim that ownership. I own my thought processes. I've willed my identity into existence by shaping it via small choices every day. I believe our differences hinge on whether a physical system is capable of causing its own internal changes. I believe a properly designed complex system is capable of that.


      Delete
    56. @ Don Jindra


      " 'There are more things in heaven and Earth, Horatio than are dreamt of in your philosophy.' "


      I almost quoted that to you but decided that Shakespeare's not relevant here. :-)


      "The delta_t time is indeterminate. It looks like you are asking me to explain why something is indeterminate."


      Nope.


      "OTOH maybe you're asking how indeterminate individual states of decay could lead to a highly predictable decay of the whole"


      They don't. A decay interval measurement could be set up to determine whether or not a thermonuclear explosion is set off depending on whether or not the interval is greater or less than the median interval for the quantity of radioactive isotope that is being watched. We set the experiment and watch a single interval, already knowing the median either by calculation knowing the properties of the isotope and how much of it is being watched or by simply measuring the median by measuring a bunch of decay intervals before setting the experiment and measuring the next, single, highly consequential, decay interval.


      "the implication being that the small cases of randomness average out to irrelevancy?"


      No. The Hurricane hit New Orleans. It was the result of a highly complex, very large quantity, of individual, microscopic,  indeterminate atmospheric events that averaged out to the quite relevant, unpredictable result that New Orleans got side-swiped.


      I am trying to show you something.


      "A brain is not a Turing machine."


      Indeed it is not. More to the point, it cannot be simulated on a Turing machine because of the random neural activity. What I wish to show you is that the random activity in the ensemble could impact the brain's "ordinary" neural activity in a way that changes what the "ordinary" activity does.


      I have not even begun to do this.


      I have, so far:


      1 -  tried to show you that a single quantum event can be amplified in a way that uncertainty derived therefrom can rule very large events (the decay interval experiment)


      2 - that averages of many events do not necessarily "return to the boring median" but can have big consequences (the hurricane).


      3 - that there is a finite (non zero) amplitude that the random activity in the brain could cause its owner to dance a jig.


      Now I can anticipate a comment that this would just be an extremely unlikely random behavior that could not be intentional, just unpredictable, but …


      … the best is yet to come.


      Have you followed this so far?


      Tom Cohoe

      Delete
    57. @ Don Jindra:

      Can a physicalist claim ownership of his/her thought processes? I can and do claim that ownership.

      So you're telling me that matter can *choose* what Laws of Physics it is going to obey?

      I believe our differences hinge on whether a physical system is capable of causing its own internal changes. I believe a properly designed complex system is capable of that.

      There's no design in Nature for the physicalist. Just an impersonal process of "random" mutation + natural "selection".

      Delete
    58. Don,

      I am quite confused by this comment. I think we are talking past each other, especially when it comes to your remarks on "technical vs. logical possibility." I frankly don't understand your point there at all.

      To rephrase and elaborate on what I said initially: the *entire point* of the p-zombie experiment is that the p-zombie is physically identically to a human. That's precisely how it attempts to show the physical brain doesn't explain the existence of consciousness. If the p-zombie weren't physically identical to us, then the thought experiment wouldn't even in principle pose a problem for physicalism. We might try to imagine some p-zombie living in world with entirely different laws and entities that can do everything we can do without the conscious experience. "Big whoop," says the physicalist, "but our claim is that in our universe, given its physical laws/entities, a physical brain like ours is conscious." The entire exercise would be pointless from the get-go.

      OTOH, your take on the p-zombie experiment seemed to be that even if nature via evolution could produce a non-conscious organism that could do everything we do, it might be more inefficient at it and hence the conscious system was selected for instead. This just doesn't track with the point of the thought experiment whatsoever -- it's an entirely different question.

      From here we have the two main points raised in your response: 1.) "Begging the question" 2.) "Technical" vs. "Logical"

      1.) Here's the p-zombie argument:

      P1 - "If it's logically possible for a being to be physically identical to a conscious human and yet not be conscious, physicalism is false." (Unobjectionable, as physicalism is preciesly the thesis that consciousness is entirely physical).

      P2 - "Such a being is logically possible."

      c - "Physicalism is false."

      The only way we can beg the question here is if we just assume P2. But that's the entire crux of the debate. I motivate P2 based on how everything from modern neuroscience tells us that we can explain the structure and function of the brain in entirely third-person neurochemical/biophysical terms without ever referring to a first-person awareness at all. The only reason we infer that such a subjectivity exists in others, I'd argue, is that each of us extrapolates from our knowledge of our own conscious awareness. This is what I was getting at with the "Figure #4" example -- on physicalism, nothing even in principle about our mental lives isn't exhaustively captured by talking about neurons firing and all the rest. And so if the physical doesn't explain the existence of consciousness, physicalism is false.

      "But the jury is out on whether science can show how these physical processes and structures necessarily entail consciousness," you say. Fine, I never assumed otherwise. All I'm saying is that we have some good reasons so far to doubt this will happen. Other people might want to argue that it's an insurmountable logical hurdle -- I'm not doing that.

      2.) Quote: "I deny the technical practicality of p-zombies. Consciousness is a technical problem. It's not a logical problem. A mere definition cannot change the problem from technical to logical, or from nature to storytelling."

      I genuinely don't understand this distinction. Everything you said in this section boils down to the following: "there may be unknown reasons why a brain like ours that physically does what it does, and physically is what it is, must be conscious." All the rest about "logical" vs. "technical".... it just comes across like replacing words to make what we're talking about sound more empirical than conceptual. Substantively, I don't understand what difference it makes what word we choose. The only reason I even included the modifier "logical" was to try to avoid the sort of misconception I flagged earlier.

      Delete
    59. @Don

      "You expect to unite and organize size, color, shape etc. but I don't know what you mean."

      You defines material as that what can be detected by our senses or instruments but them go on to talk about material substances, immaterial substances etc. A substance, material, immaterial or of another type, would be something that is the "owner" of the characteristics that we see with our senses, like "this brownness is that dog color". On a ontological level, it is substances who organize the qualities in separate things and on a phenomenological level it is the concept of substance that help us separate the data of the senses on diferent beings, detect patterns etc.

      The trouble with your epistemology it that our senses and instruments can't really show us any substance, material or whatever, all they show are several types of qualities and it is our minds that structure sense data using(along with other things) the concept of substance. Knowing all this, i don't see any grounds on your epistemology to talk about substances, for your own definition of material seems to exclude all that is not got directly by the senses. This also means that there is no reason to talk about materialism, for it pressuposes a particular type of substance.


      "As far as I know, both Mill and Hume were empiricists who believed knowledge is gained only through experience of the natural world. You imply they also believed there may be nothing of substance driving that experience. What text are you referring to?"

      I can't remember a exact book from Mill but one can't go wrong with young Hume book A Treatise in Human Nature and also with his A Enquiry Concerning Human Understanding. These are very good books to cure one of naive empiricism.

      I'am talking about that because you seems, seems, afflicted by this disease. From your post here i can cite you talking about how something who can't really be tested is not important, as if you were not already on philosophy land since your first post, talking of future experiments showing that we can or not simulate minds, as if we had any way of detecting directly a mental life on the robots and not just infer things based on neuron activity and stuff, and talking as if things like communication showed that other minds are real etc.

      If one really push to the limit what our senses and instruments actually say, one finds hard to believe that it is true, as your average empiricist suppose, that empirical evidence say much on philosophy of the mind. My original post was precisely to to see if you are really a victim of naive empiricism or not. For if you is then it would explain why the discussion here seems a bit pointless. If yes, i recommend some Hume and things like the myth of the given and Quine indeterminancy of translation to try to help visualize this talk more clearly.

      Delete
    60. @Don Jindra:

      Regarding your hypothetical future "conscious" machine: could we create a conscious machine and deceive it but making it believe it has free will?

      Delete
    61. Tom Cohoe,

      We agree a brain is not a Turing machine and there is no good reason to believe a Turing machine can simulate the brain's activity.

      We agree there are butterfly effects.

      We agree complex, dynamic systems behave in ways that can never be predicted with perfect accuracy, and the further away we get in time the less accurate the predictions become.

      We agree the brain has some random activity. I mean, some of my dreams are pretty random.

      So where are you going?


      Delete
    62. UncommonDescent,

      "So you're telling me that matter can *choose* what Laws of Physics it is going to obey?"

      No, laws of nature are not broken. I'm saying even simple clumps of matter can 'select' among several alternatives. There isn't necessarily one and only one alternative. Brains are highly complex. They are not billiard balls or pinball machines. I assume you agree with that much. I don't agree with your assumption on the limits of physical machines.


      "There's no design in Nature for the physicalist. Just an impersonal process of 'random' mutation + natural 'selection'."

      Don't call it 'design' if you don't want to. But there is little doubt in my mind that there is plenty of organization in nature. This organization looks designed and/or is very functional. I don't see this happening as a sequence of random events. For example, water flows downhill. It can't follow any random path from mountain to ocean. It mostly follows paths of least resistance. Feedback keeps evolution from following random paths. To characterize nature's 'choices' as random doesn't present a complete picture.

      Delete
    63. Alex,

      "To rephrase and elaborate on what I said initially: the *entire point* of the p-zombie experiment is that the p-zombie is physically identically to a human."

      I deny this is possible. The p-zombie -- a thing that looks like a human but has no consciousness -- could never behave like an actual, normally functioning human being. So I deny the 'experiment' has any validity. It does not represent a possible reality. I will elaborate shortly.


      "If the p-zombie weren't physically identical to us, then the thought experiment wouldn't even in principle pose a problem for physicalism."

      Exactly. This is my point. The thought experiment doesn't in principle pose a problem for physicalism. It assumes from its very start that consciousness is irrelevant. We can simply imagine it away. So it's no surprise that the conclusion is that consciousness is irrelevant. It began that way.

      The crux of the issue is here:

      P1 - "If it's logically possible for a being to be physically identical to a conscious human and yet not be conscious, physicalism is false."

      My objection, as stated above, is that it's probably not technically possible for a physically identical p-zombie to exist. The supposed logical possibility is irrelevant. So based on the technical flaw in the thought experiment, I have no reason to follow the logic after that.

      Further elaboration:

      The p-zombie thought experiment claims the p-zombie is "logically" possible. But I claim thought experiments like this merely show the limitation of logic. We can agree that p-zombies do not exist in nature. The question becomes, can they exist? Logic cannot answer that question. When logic depends on something that may be physically impossible, the truth of the conclusion is in doubt.

      To counter the p-zombie thought experiment, I could create a p-Data thought experiment. We can imaging a robot like Data on Star Trek.

      So,

      P1 - If it's logically possible for a robot to be physically identical to a human and exhibit conscious behavior, dualism is false.

      P2 - Such a being is logically possible.

      c - Dualism is false.

      IMO, there is no merit to such a logical proof, yet it's structurally the same proof as the p-zombie proof.

      Similarly, if I create a thought experiment that starts with a supposed physically identical copy of a human who can jump over the moon, what logical conclusions can be drawn about muscles and bones? Surely none that say anything about reality.

      When you say you don't understand my distinction between technical problems and logical problems, I don't understand how you can't see the distinction. I have expressed my skepticism about using logic to draw sweeping conclusions many times on this site. It can mislead people who put too much faith in it. That's what I think the zombie argument does. It misleads because it starts by ignoring reality then attempts to draw conclusions about the reality it ignored from the start. As a truth-discovering methodology, it's flawed.


      Delete
    64. @Don Jindra:

      I'm saying even simple clumps of matter can 'select' among several alternatives. 

      Well, what you're saying is devoid of logic.

      First I would like to know why the use of scare quotes when you are speaking of the 'selecting' capabilities of matter.

      Second, the neurochemicals that are released into the synaptic gap and that trigger neuronal activity are forced to obey the same set of Physical Laws that the water you mention. Quoting you: For example, water flows downhill. It can't follow any random path from mountain to ocean. It mostly follows paths of least resistance.

      So if water is not 'free' to choose its path, neither are the neurochemicals released into the synaptic gap, unless you are trying to tell me that synaptic gaps are magical places in the Universe.

      Third, the paragraph I quoted from you is a clear evidence that there is telos in Nature. More on this:

      To characterize nature's 'choices' as random doesn't present a complete picture.

      True. That Nature is ordered and tends to certain outcomes is very dear to the heart of the Aristotelian-Thomist proponent. But the A-T proponent is justified in proposing such a picture, because the A-T  allows for formal and final causation (they are intimately united, you can not have one without the other).

      But the physicalist/atheist can not allow for formal causation, because in living things, the form of the body is the soul that you despise so much, and allowing an immaterial entity into your metaphysics would destroy it.

      So the physicalist/atheist realizes there is order in Nature, but can not explain why it is so. And that's when he appeals to those amorphous 'Laws of Nature' that are just 'brute' facts.

      "Well, the Universe follows these 'Laws' but we can not explain where they do come from, it's just that they happened to happen". A bad idea from a bad metaphysics which allows Universes 'poofing' into existence for no reason.

      And last but not least:

      But there is little doubt in my mind that there is plenty of organization in nature.

      Same thing happens to me. But the  theist has God (the summum of rationality and lawfulness) on his side and your physicalist side has, well... a bunch of 'dumb' matter. What an immense difference does it make.

      Delete
    65. Talmid,

      "The trouble with your epistemology is that our senses and instruments can't really show us any substance, material or whatever, all they show are several types of qualities and it is our minds that structure sense data using (along with other things) the concept of substance. Knowing all this, I don't see any grounds on your epistemology to talk about substances, for your own definition of material seems to exclude all that is not got directly by the senses."

      I do not deny the ability of the brain to organize sense data into a model of reality. In fact, that's one of its main purposes. Even an ordinary micro-controller system is designed to organize sensed data. I'm fully aware that we sense the world indirectly. So your complaint is of no concern to me.


      I've read "A Enquiry Concerning Human Understanding." I thought Hume's discussion of cause and effect was pretty accurate. But like most philosophers he carried things too far in the second half. Nevertheless, in what way do you think he 'cures' naive empiricism? I assume by naive empiricism you mean the search for falsification.


      "From your post here I can cite you talking about how something that can't really be tested is not important"

      I'm concentrating here on truths that are independent of our human perspective. We shouldn't be using our human perspective to muddy up those truths. That's what falsification should test -- truths that are independent of us. It's true that I think Grace Kelly was beautiful. A handful of people will say otherwise. But I'm not too concerned about falsifying that statement because it's not meant to be a statement outside human perspective.


      "as if we had any way of detecting directly a mental life on the robots"

      Why wouldn't we? We don't have the ethical concerns we do for humans. So we can probe the robot's mental states any way we choose. OTOH, if we become concerned about the ethics of probing a robot's mind, it means we are convinced of its consciousness and identity.


      "talking as if things like communication showed that other minds are real"

      Communication can show other minds are real. Communication has convinced me my wife's mind is real.



      Delete
    66. UncommonDescent,

      "could we create a conscious machine and deceive it but making it believe it has free will?"

      Interesting question. I don't know.

      Delete
    67. Don Jindra,

      Let's try a different tack. You clearly think consciousness is just a physical process. But if I'm understanding you rightly, you want to say that it's not all physical processes that possess consciousness, since that way lies panpsychism. So let's take physical process X and physical process Y. X has consciousness and Y doesn't. What in your view is the difference between them?

      Delete
    68. @ Don Jindra ,


      I am travelling today so I hope I can find time to give you something else to digest. I will be a bit sloppy with the distinctions between "ordered", "disordered", and "random", to save time. These distinctions are, in themselves quite interesting, especially in the way that they are analagous to other distinctions, but for now, at least, if you will allow this elision of distinctions, I will proceed.


      The first thing I wish to demonstrate is something that you, as a firmware engineer, should readily understand. It is that a random sequence can transmit instructions that are preconceived and intentional. As the sequence is truly random, someone who tries to tease out the transmitted intent has an impossible task … this is what I intend to demonstrate.


      An ordinary computer cannot output an unbiased random number. That is why we properly call the "random" number output of a computer a "pseudo-random" number. The longer the pseudo-random sequence, the more biased the pseudo-random number because the fraction of all possible pseudo-random numbers that the _computer_ can generate becomes a smaller and smaller fraction of all numbers of the same length, as the length grows. A truly random number could be any one of all numbers of the same length, with equal probability (to be unbiased).The fraction mentioned above approaches 0 as a limit as the length of the number approaches infinity.


      To clarify here, by "number" I mean a sequence of binary digits - a sequence representable as a sequence of "0" and "1". The "length" of the number is the "length" of the sequence.


      Let us consider the XOR operation between two single digit numbers. We have:


      1 XOR 1 -> 0

      1 XOR 0 -> 1

      1 XOR 0 -> 1 and,

      0 XOR 0 -> 0


      We also have, where A and B are single digits:


      (A XOR B) XOR A -> B and

      (A XOR B) XOR B -> A


      (do the operation twice in a row and you get your starting digit back. It's symmetrical as 

      A XOR B = B XOR A)


      I do not have time to go any further with this today, but you might notice that I am setting up what used to be called a "one time pad", 


      https://en.m.wikipedia.org/wiki/One-time_pad


      , known and used since 1887.


      Tom Cohoe

      Delete
    69. @Don

      "I do not deny the ability of the brain to organize sense data into a model of reality. In fact, that's one of its main purposes. Even an ordinary micro-controller system is designed to organize sensed data. I'm fully aware that we sense the world indirectly. So your complaint is of no concern to me."

      You say that you are only interested on the empirically given and now admits(i suppose that on this quote you are agreeing with me) that both never give us any kinda of substance,casuality etc. Ergo, you have no ground to these. Come on man, that is exactly Hume point. Naive empiricism is precisely this erroneous combination of empiricism with belief on concepts that are never given by the senses that David tried to refute.


      "I'm concentrating here on truths that are independent of our human perspective. We shouldn't be using our human perspective to muddy up those truths. That's what falsification should test -- truths that are independent of us. It's true that I think Grace Kelly was beautiful. A handful of people will say otherwise. But I'm not too concerned about falsifying that statement because it's not meant to be a statement outside human perspective."

      Very few truths are capable of being falsified(if even scientific ones can is a open question, but that is not important) Take for instance things like:

      - there is change

      - human thought can have intentionality

      - living beings are possible


      Just the act of trying to set-up a possible way of falsifying these is only possible if they are true, ergo, they can't be false. This mean they are all unfalsifiable and true. They being unfalsifiable does not mean these can't be trusted, they are actually way more certain that any falsifiable truth.

      You seems to have a bad vision of how a metaphysician works, no doubt thanks to stupid thought experiments like philosophical zombies, and it is probably why you seems to contrast it with falsification. The thing is that philosophy is not stuck on appeal to "intutions" but we can use way better methods, as the truths i mentioned before show. A good post grom Dr. Feser on the subject is this one: http://edwardfeser.blogspot.com/2020/01/the-rationalistempiricist-false-choice.html?m=1

      "Why wouldn't we? We don't have the ethical concerns we do for humans. So we can probe the robot's mental states any way we choose. OTOH, if we become concerned about the ethics of probing a robot's mind, it means we are convinced of its consciousness and identity."

      How would we probe the robots mental states? The best you can do is look at what functions like a brain to they, but how would we go from the third-person brain to the first-person mind? Would you not agree that a completely deaf person that knew exactly how your brain functions while listening to music and tested you with all possible tests would not be capable of knowing your experience as you do?

      I do not even mean that as a argument for dualism(as i do not see consciouness as immaterial), my whole point is epistemological. What i'am trying to show is that materialism is not more empirically supported that any of his rivals, it is a metaphysical position and nothing besides.

      "Communication can show other minds are real. Communication has convinced me my wife's mind is real."

      My mind is real too!

      The point is that the communication you have with your wife it not enough to say that it "shows" that her mind is real, sense data alone wilk not do it because you can't really observe her mind, only exterior behavior that make you infer that she has a mind.

      Don't get me wrong,you are as rational as you can be in infering that she has a mind, but it is not empirical data alone that seals the deal. From only the empirical, she could be a complex robot, a dream etc, so on believing that she has a mind you are(correctly) going beyond what your senses can give you, at least if we take the senses on a empiricist view.

      Delete
    70. @ Don Jindra,


      I assume now that you know the principal of the one-time-pad. Here, now, we have sequences of binary digits, represented as strings of 0's and 1's. I will call them:


      "K"  (think of "KEY"),

      "PT" (think of "PLAIN-TEXT"),

      "CT" (think of "CIPHER-TEXT"),

      "SK" (think of "SUSPECTED-KEY").


      The strings are of length n. I will simplify this here by writing *XOR" as "+".


      K is first produced by using a method based on the uncertainty principle such as I have given. Each binary digit of K is unbiased and random, it having a 50/50 probability of being a 0 and a 50/50 probability of being a 1. I will say that it is a 50/50 digit. It is simple to see that if K consists of 50/50 digits then K must be an unbiased random sequence ("disordered" is more correct because K is finite - length n - but we are ignoring this distinction in this writing, as previously discussed, for brevity).


      The ship captain takes a copy of K and locks it in his safe. The only other copy of K is in the possession of the admiral on shore who does not want the captain to know his orders until he is distantly at sea. The orders are written as the string PT in some widely known digital code such as ASCII. 


      Then CT is produced as PT+K->CT and CT is sent by radio to the captain. The captain recovers PT, his orders, by CT+K-->PT.


      Much I elide here for brevity but will fill in at request.


      The digits of CT are all 50/50 digits and CT itself is an unbiased random sequence. This means that the captains orders are not in CT. CT is just a random sequence. Anyone who has intercepted CT cannot extract PT from CT because PT depends on K which the interceptor does not have. For suppose the interceptor suspects the key is SK and he "confirms" this in his mind when he tries CT+SK->SPT (think of "suspected plain text", ie suspected captain's orders) because SPT is a fully intelligible ASCII string.


      However, no matter what string SPT is, SK, "confirmed" by the intelligibility of SPT, a version exists of SK for which CT+SK->SPT and that version is SPT+CT->SPT. In other words, that the intercepted CT XORed with the SK, the suspected key gives an intelligible result is just plain chance, as there is a version of SK for every conceivable intelligible sequence of length n, and so the "confirmation" is just a delusion.


      So CT does not have PT in it. CT has, instead, all possible intelligible sequences of length n in it, which means that it doesn't meaningfully convey the intended PT. CT is just an unbiased random string of length n, one element of the 2^n elements of the set of all binary sequences of length n. Furthermore, the interceptor, who has been looking for PT by going through every version of SK ( he has a very fast computer) and looking for the production of an intelligible string as "confirmation" that SK is K, since he has been using the XOR method has to know that he is doing nothing better than guessing PT, the actual orders. He would do just as well to hire one of those people who advertise themselves as "psychic mediums" and have them write down an intelligible guess as to the captain's orders.


      In brief, CT does not contain the orders, nor does K. The orders, preconceived in the sense that they are conceived before CT is generated (but after K, so K does not contain the orders either).


      So we have an unbiased random string, CT, causing the captain to do what the admiral has decided he wants him to do.


      I believe that this is what I was to show, that a random sequence can elicit preconceived intentional activity.


      I left out showing that CT is an unbiased random sequence.


      Tom Cohoe.

      Delete
    71. @ Don Jindra,


      I must correct an error in the above caused by my confusion (I get confused when I am writing very easily :-) ) This is the paragraph with the error:


      "However, no matter what string SPT is, SK, 'confirmed' by the intelligibility of SPT, a version exists of SK for which CT+SK->SPT and that version is SPT+CT->SPT. In other words, that the intercepted CT XORed with the SK, the suspected key gives an intelligible result is just plain chance, as there is a version of SK for every conceivable intelligible sequence of length n, and so the 'confirmation' is just a delusion."


      Specifically, "SPT+CT->SPT" should be "SPT+CT->SK". That, I hope, should make more sense.


      Further up, I had *XOR", which should have been "XOR".


      Tom Cohoe

      Delete
    72. UncommonDescent,

      I used scare quotes around 'selecting' to stay away from an anthropomorphic connotation.

      "So if water is not 'free' to choose its path, neither are the neurochemicals released into the synaptic gap, unless you are trying to tell me that synaptic gaps are magical places in the Universe."

      Let me try to be clear about what I'm saying. Water does not have one and only one path it can follow from mountain to ocean. There are billions of possible paths it could take which would not defy the laws of physics. The path we see is the path taken because of a combination of physical law and randomness. But there were even more paths it could not have taken. Those paths would have defied the laws of physics. Laws of physics put constraints on matter but they don't force matter into one and only one option.

      You want to call this constraint telos. IMO, that's an anthropomorphism. I think "following paths of least resistance" is more accurate. I do not deny nature has paths of least resistance, which we could call laws of physics. But I do not want to imply intent or purpose.


      "because in living things, the form of the body is the soul that you despise so much, and allowing an immaterial entity into your metaphysics would destroy it."

      I do not despise the soul. I have often complained that this or than in our culture (e.g, rap music & 'reality' TV) is destroying the souls of some people. We look at the concept differently, that's all. I won't allow an immaterial entity into my metaphysics for one reason: It explains nothing and never can.

      Both 'laws of nature' and The 'immaterial soul' can be considered brute facts. The difference as I see it is that laws of nature are understood to be provisional. We admit we could have them wrong. We admit that because we know they are models based on limited observation. But there is no observing of an immaterial soul.

      Btw, I do not believe the universe 'poofed' into existence. The stuff of the universe is eternal. It eternally changes form.


      "the theist has God (the summum of rationality and lawfulness) on his side and your physicalist side has, well... a bunch of 'dumb' matter. What an immense difference does it make."

      What difference? I'm serious. What positive difference does it make?



      Delete
    73. DrYogami,

      "So let's take physical process X and physical process Y. X has consciousness and Y doesn't. What in your view is the difference between them?"

      I don't know. I wish I did but I don't. It is okay to admit we don't yet know everything.

      Delete
    74. Tom Cohoe,

      I don't understand your goal with the encryption/decryption example. The message, PT, is split into two pieces, CT and K. The token received second, CT, is not the actual message, PT. To become the complete message the two pieces (CT & K) are reassembled. Both CT and K are random sequences when considered separately but not once reassembled. How do you see this as relevant?

      Delete
    75. Talmid,

      "You say that you are only interested on the empirically given and now admit ... that both never give us any kinda of substance, casuality etc. Ergo, you have no ground to these."

      We don't have to *be* the chair before learning some things about it. Indirect sensation is not 100% unreliable. You're creating a false dichotomy.


      "but how would we go from the third-person brain to the first-person mind?"

      The question that haunts us here is, Can a machine have conscious experiences and how would we know? We can ask a person, What was it like to skydive? His description of his experience may be a simple "Like wow!" In that case he's no help. But some people can describe their experiences in vivid detail. We go to movies and read novels in search for those indirect experiences. When the novel or movie is great, we do get emotional reactions that mirror a first person account. When a machine can experience and convey that experience in that way, I don't think there will be much doubt that the machine had a conscious experience. At that point we could probe its mechanics to see what's going on, if we didn't already know that in the design phase.


      "Don't get me wrong, you are as rational as you can be in inferring that she has a mind, but it is not empirical data alone that seals the deal."

      Though experience is extremely important to us, I don't hold the 'blank slate' view. We're designed to understand reality in a way that benefits human biology. I know our brains filter data. We have builtin, confirmation bias. We don't see a flower like a honey bee. We have a nature. That nature guides us through the environment as much as experience does. You're taking the position that a biological nature can't help us infer from experience. But I don't see why not. It seems like a practical way for our brains to function.


      Delete
    76. @Don Jindra:

      Interesting question. I don't know.

      And how do you know you are not that type of machine? Believing you have free will while in fact you are being deceived?

      Delete
    77. @ Don Jindra,


      "I don't understand your goal with the encryption/decryption example. The message, PT, is split into two pieces, CT and K. The token received second, CT, is not the actual message, PT. To become the complete message the two pieces (CT & K) are reassembled. Both CT and K are random sequences when considered separately but not once reassembled. How do you see this as relevant?"


      The example of the one-time-pad is relevant in one important way. It shows that a rule does not exist that a random sequence of events is not purposeful in a preconceived and detailed way. The one-time-pad (OTP), because of its symmetry, is easy to understand for demonstrating the non-existence of such a putative rule. But that this is shown here for the simple and symmetric OTP only, does not allow a conclusion that random sequences that are not part of an OTP equivalent system, are meaningless and without purpose. The common idea that randomness is cold and dead is broken because the OTP is a counterexample.


      So randomness in the brain could be intentional and detailed, even if it is truly random. Mind function could be embedded in it and you could not prove that it isn't. Furthermore, continuous, detailed, neuron level observation of the whole brain would continuously destroy the growing superposition making conclusions presumptuous indeed, and likely to be wrong.


      The same goes for biological evolution. It could be intentional and that too could be impossible to demonstrate.


      So many other phenomena that atheists have described as "now understood" actually are not because amplified microscopic randomness makes them unpredictable in principle. The Titanic sank because it struck a big iceberg - understood, not a God mediated event. Sure, sure, there were a whole slew of unpredictable factors in that sinking, from the mind mediated behavior of people to the weather that grew the ice so that the iceberg calved off when it did and followed a track that led it to meet with the Titanic on the fateful night.


      It's actually this way with virtually everything.


      At least - and this is the important thing - you can't prove that it isn't. Science hasn't made one inch of progress in proving that God stands on shrinking ground, or that we are on the verge of understanding mind, and it ought not to, for it isn't the purpose of science.


      Furthermore, it can be illustrated that there are more random ways to achieve an end than there are ordered ways to achieve that same end - i.e. random behavior is more powerful than ordered behavior, and order is all that science can extract from observations.


      Tom Cohoe

      Delete
    78. @Don

      What i'am trying to say is: every position is very "logical" on his arguments and evidence, even the so-called empirical ones, like materialism is said to be by some of its defenders. Materialism is not more suggested by empirical data than other views like idealism, hylemorphism, pantheism etc.

      While analytical philosophers take for granted a certain view of reality, it is just a view among others. If you want to know which is right, you will have to step on the land of the unfalsifiable and do philosophical arguments.

      Delete
    79. @Don Jindra:

      The path we see is the path taken because of a combination of physical law and randomness.

      What do you mean by 'randomness'? Do you mean that there is an epistemological limitation on our part?

      I do not deny nature has paths of least resistance, which we could call laws of physics. But I do not want to imply intent or purpose.

      Neither do I. From an A-T perspective, telos in Nature does not imply 'conscious purpose' of the substances undergoing change. It's not that water 'thinks' that it wants to achieve an end. It's that water, due to its nature, gives a determinated range of outcomes (for example being fluid between 0-100 degrees and following the mentioned paths of least resistance when travelling from mountain to ocean).

      I've noticed that for materialists, the concept of telos is incredibly hard to wrap their heads around, because you are obsessed with the 'lack of purpose' in Nature. Which is curious because for lots of materialists, the 'purpose' of their lives is to convince others to their religion and to create an scientific-atheistic Paradise on Earth. Paradoxical indeed :)

      I won't allow an immaterial entity into my metaphysics for one reason: It explains nothing and never can.

      Wrong. The immaterial soul can ground the permance of your identity over time. Which is something that your physicalism fails miserably to achieve. Since we are material beings and matter is constantly subject to change (including the matter of the brain), it's not possible for you to explain how the self remains constant since birth to death.

      Btw, I do not believe the universe 'poofed' into existence. The stuff of the universe is eternal. It eternally changes form.

      So you're contradicting science which says that the Universe began existing 13.8 billion years ago.

      What difference? I'm serious. What positive difference does it make?

      Well, there's nothing strange for creatures that have been ahem   *created* in the image of the summum of Rationality and Lawfulness to be able to use their rational faculties to understand the Universe and for them to believe that there is order and purpose to be found. No surprises here. Order begets order, rationality begets rationality, lawfulness begets lawful behavior.

      Not so much for the physicalist who has to explain why an unintended hominid in an uncaring and impersonal Universe should be able to trust his cognitive faculties and to expect to find order and lawfulness out there. Based on what should we expect that behavior from reality? Who's to say that reality has to be bound by order and that our tiny brains can unwrap it? No reason at all from the physicalist side.

      So, as I mentioned, it makes a whole world of difference. An unbridgeable one indeed.

      Delete
    80. Don Jindra,

      I don't know. I wish I did but I don't.

      As I recall, you've said similar things in the past, like that you wish you knew how concepts could be 'material'.

      This increasingly looks like an article of faith on your part. You seem awfully wedded to the materialist thesis for a reason I just can't fathom. I'm not settled on any metaphysical schema just yet, but the materialist paradigm just seems implausible from the get-go.

      Delete
    81. Talmid,

      "While analytical philosophers take for granted a certain view of reality, it is just a view among others. If you want to know which is right, you will have to step on the land of the unfalsifiable and do philosophical arguments."

      I won't take a relativistic stance and I doubt you're taking one either. I do think some mental/philosophical models are better than others. And by better I don't mean more pleasant. I mean they can demonstrate results, they seem to fit reality more. There are those who reduce all such judgments to values. I mean, why should we value 'reality' and practicality above our emotional or hedonistic needs? Why should we care if a billion people live in hardship as long as the best among us prosper? While there may be some truth to these positions, I have a hard time believing some people believe what they claim to believe. It's easy to curse medicine until one's own life is at stake. I do not limit myself strictly to ideas we can falsify. If I gave that impression I didn't mean to. My position is that knowledge about reality is unreliable if it's framed in a way that can never be falsified. It may not be false. But how would we know if it's true or false? I acknowledge we all have to make first assumptions. I don't claim empiricism can prove empiricism. But I do think we can follow the trail of where our first assumptions lead and if they lead to only strife and/or navel gazing then what's the good in that?

      Delete
    82. @Don

      "My position is that knowledge about reality is unreliable if it's framed in a way that can never be falsified."

      Well, i understand your fear of the methods that philosohy uses being worthless, no building fails down if a philosopher is a idiot and the area has no consensus in anything. If i had no Church saying that the human mind is capable of finding a kot of truth, i probably would worry about that a lot more.

      The part on your reasoning that makes me react is that you forget about this skepticism when defending materialism. Maybe reality looks that way to you, that is one thing, but empirical evidence can't say much directly about it being right or wrong, at least not on a scientific setting. So i think that you should be a bit more prepared to take serious writings like Dr. Feser post that go and say "this model we have of matter is is only one option and it looks like it is false because so and so, we should drop it", even with no empirical data.

      My objective is more to try to let your skepticism be a bit more universal, that is why i've been pushing the role of our pressupositions on a lot of "empirical" data. If you don't assume that materialism is true, are there good reasons to think that it can solve the problems it has today?

      And a tip, if you are interested in falsification, take a look at things like miracle claims*, near-death, experiences, mystical experiences etc around the world. I saw a lot of materialists who say they want empirical evidence but never serious look after it, which is sad. If the world shows nothing strange, them the materialist view can claim some confidence, but if things are wacky...

      That is probably your best bet at getting empirical evidence that could be useful here.


      *we catholics have ones that are studied by people well-paid to be skeptics appearing even today. It is also cool to remember that the christian faith has a lot of falsifiable claims like the Exodus, the ressurection of Christ etc, why not take a look at these?

      Delete
    83. UncommonDescent,

      "And how do you know you are not that type of machine? Believing you have free will while in fact you are being deceived?"

      I know because I know myself. Your question was if we could create a conscious machine and through deception make it believe it has free will. In my case, I would have to be the deceiver.

      First thing to consider is consciousness itself. What is it? I don't know. Maybe it's partially the "what is it like" to make a choice. It's certainly more than the ability to select among possible simple options.

      The second thing to consider is what is free will? I don't like that term. I'd rather call it will. Will to me is choosing the harder option. If we choose the easier option every time, we wouldn't have will. Every human is face with tough choices. A hedonist probably rarely chooses the tougher option. He goes for immediate gratification. A person with will chooses the option that may never pay off. But he has hope it will pay off. The payoff is not necessarily more future pleasure. It could be self-satisfaction, or benefit of others, or even a dogged persistence in a self-identity -- fostering ideals like honor, honesty, piety and fidelity.

      I know in my life I've chosen the tougher path many times. And this is the primary reason I know I have will.

      I don't know if a machine could be manipulated in a way that the tough choice always seems like the easy choice. And if it was tricked into thinking this, does that really make the tough choice the easy choice? Or is this better described as an ideal form of will? 'Free will' would then be a kind of deception (though not the deception a Sam Harris thinks) -- a self-generated deception into making some tough choices into easy choices.



      Delete
    84. DrYogami,

      "You seem awfully wedded to the materialist thesis for a reason I just can't fathom. I'm not settled on any metaphysical schema just yet, but the materialist paradigm just seems implausible from the get-go."

      I am wedded to the materialist thesis because nothing else makes sense to me. For example, I could never understand how Plato's forms were supposed to have anything to do with reality. All non-materials explanations strike me as variations on Plato.

      I don't deny materialism is, at root, a faith. But I do deny all faiths are equally plausible.

      Delete
    85. @ Don Jindra,


      "The second thing to consider is what is free will? I don't like that term. I'd rather call it will. Will to me is choosing the harder option."


      Better to call it free will. The harder option is not always the right option. Some habits are better kept. Breaking a habit is the harder option. You don't want to break all your habits do you? Do you want to quit eating?


      In my strictly agnostic days, I thought, like you, that there was no way to know which of directly opposed faiths could be true. In your case this is materialism versus something you don't even want to acknowledge ("All non-materials explanations strike me as variations on Plato")


      Reading Aquinas, I came to realize that sense could be made of belief in God, that it is in fact very reasonable. I also came to realize that intellect, much more than consciousness, actually is intractable in materialist terms (I also, like you, had once thought "I am aware, therefore I am" was the big problem, not "I think, therefore I am"). I made the free will choice of joining the Catholic Church 4 years ago, after decades of wandering in seemingly intractable agnostic limbo. The choice was easy, but not until after I had done the hard work of reading the Summa and thinking about the difficult matter (to a modern mind) of understanding Thomistic hylomorphism. Reading the Summa was a matter of what we Christian's call "grace". I'd had a copy that my dad had left me just sitting around for many years. One day, seeing it, I just thought, "what is it about Aquinas that makes so many people talk about him", so I picked it up and started working at it. It just amazed me. You might call it luck or chance, but naw, I was meant to read it. The various random (yes random) factors in and around my life put it into my hands when other random factors had made me ready for it.


      Materialism is a habit. Breaking it would be the hard option for you. But maybe you should break it. Let your intellect work on that idea.


      Plato didn't completely think through the idea of forms, so characterizing the Aristotelian/Thomist/Christian idea of forms as Platonic dualism isn't fair.


      The idea of a form is very simple. The table you sit at is a real table because it has the form of a table (if it didn't it wouldn't be a table) and because it is composed of particular matter, which is what makes it the individual real table you sit at. The form is not the table. By itself the form is just an idea in a mind or intellect (not some Platonic "place beyond heaven"). The specific matter that makes up your table is also not, by itself, a table. It is whatever it has the form of.


      Do you not think your table really exists?


      I know this will not satisfy you, and it shouldn't. But it's all I'm going to say about it.


      You would have to do the harder thing and apply your intellect to the question of whether or not you should break your materialism habit - and it _will_ be a hard thing to do.


      Tom Cohoe

      Delete
    86. @Don Jindra:

      I know because I know myself.

      Sorry but nope. Unless you can solve the riddle of how, under physicalism, the "I" who is both generated and sustained by nerve impulses in the brain and which is then prohibited to have any real power to decide anything on its own because it is totally under control of the physical forces producing it, can be, at the same time, a causally efficacious individual (both statements stand in clear logical contradiction), your position explains nothing and does not deserve to be taken into consideration.

      And you are going to need far more than vague appeals to "complexity" and to some "future" science.

      Delete
    87. @ Don Jindra,


      "The path we see is the path taken because of a combination of physical law and randomness. But there were even more paths it could not have taken. Those paths would have defied the laws of physics."


      They could have followed any path without defying the "laws" of physics, but some of the random inputs occured millions of years earlier, or hundreds of millions, or even billions of years before the water flows to the ocean by a path that is ultimately wholly random.


      Tom Cohoe

      Delete
    88. UncommonDescent,

      "Unless you can solve the riddle of how, under physicalism, the 'I'..."

      What makes you think I must solve that problem before i even speak about knowledge of myself?

      But more importantly, what make you think you're in better position to generate that 'I'? Why should I take your proposed solution into consideration?

      I don't believe you think science has reached its limits. It's reasonable to believe there is a future in science and technology.

      Delete
    89. Tom Cohoe,

      A habit starts with choices. Many important decisions are first encounters with the several options. It all starts with plain will, imo.

      It's funny you brought up eating. There are a lot of things I would like to eat, but don't. Right now my biggest issue is with Cheetos, but it's manageable. I've broken a lot of bad habits over the years. For example, I used to drink a lot of iced tea loaded with sugar. Then I decided I was consuming too much sugar and stopped cold turkey.

      I've read some Aquinas. He doesn't do anything for me. You said you were in agnostic limbo. I've never been in limbo. My parents were not church goers when I was young but they did send us kids to Sunday school. I went until around 18. But I don't think I was ever a Christian because I never believed in the divinity of Jesus and I never believed in miracles. I never understood the Trinity. I never understood how Jesus could forgive sin. That seemed immoral. And I'll never forget, at around 10, being told the story of Abraham and Isaac. I thought it was awful to think a father would kill his son to prove his faith. It's not that I've never exposed myself to religious thought. I've made a point of reading a lot of it. I used to be a Diest, I suppose, until around 20. I just found supernatural explanations didn't connect to my world and I stopped seeing a need for it. I have nothing against religion in moderation.

      Yes I think a table really exists. It exists entirely of physical matter. Our deep thoughts won't add a thing to that matter.

      Delete
    90. @ Don Jindra,


      There is nothing in the "sacrifice" of Isaac by Abraham to allow a choice between faith in the God of love and atheism except where that choice has previously been made. Either is consistent within its own terms. The truth must be found by other means.


      Faith in the God of Love entails that Love does not destroy innocent souls. Abraham's faith in Love was so great that he knew through Love that Isaac would not be lost, except temporarily at most. Isaac was not lost. God is Good.


      If God is nothing, decided in advance before considering Abraham and Isaac, then the willingness to sacrifice a son over an insane faith in nothing can only make that religion seem evil.


      It's Love versus nothing.


      All these things that you couldn't make sense of are comprehended in an infinite, unbiased, random number, and that is something that you cannot comprehend as all your thoughts are just a finite subsequence of this number, which contains all possible thoughts. Your understanding of it can always grow, but it will always be a case of the finite beside the infinite. You will never prove or disprove that God is Love. Different means are needed to find the truth.


      Tom Cohoe

      Delete
    91. @Don Jindra:

      What makes you think I must solve that problem before I even speak about knowledge of myself?

      Because there is not a "you" reflecting on anything. "You" (according to the philosophical mess that physicalism is) is just the unintended creation of neuronal processes. For the millionth time: the conscious individual that is being both created and sustained by physico-chemical processes can not have any causal powers at all. The physical substrate 'decides' everything about it.

      But more importantly, what make you think you're in better position to generate that 'I'? Why should I take your proposed solution into consideration?

      Because the A-T proponent can offer a realistic view of the human person and assign causal powers to it. It's the whole person who reasons and decides and not the physical forces producing it.

      I don't believe you think science has reached its limits.

      What "science"? The one you contradict when it doesn't suit your interests? Because your thesis is that the Universe is "eternal stuff changing form" while science clearly states that it had a beginning 13.8 billion years ago (The "Big Bang" and all that).

      Delete
    92. UncommonDescent,

      "the conscious individual that is being both created and sustained by physico-chemical processes can not have any causal powers at all. The physical substrate 'decides' everything about it."

      There is no reason for me to accept that assertion. There is very good reason for me to reject it -- on empirical grounds.


      "Because your thesis is that the Universe is 'eternal stuff changing form' while science clearly states that it had a beginning 13.8 billion years ago (The 'Big Bang' and all that)."

      My thesis aligns with science. The Big Bang was not the beginning. It was a transitional phase. Even when physicists talk about a "nothingness" prior to the Big Bang, it's an equivocation. They don't really mean nothing. They still assume some law(s) of physics exist to set off the bang. Those laws are not nothing. The gracious Mr. Feser has made this argument. It's one philosophical point on which we agree.

      Delete
    93. @ Don Jindra,


      I do not know, because you have not completely replied, the complete significance to your thinking of the existence of random sequences which, although without inherent meaning, can effect the detailed intent of one mind in another.


      It is that there is no rule that what is observed to have no order, i.e. which is completely random, cannot be effecting a highly ordered result, i.e. the actual course of evolution, i.e. to your existence and mine specifically out of the very high number of results in which we do not exist under similarly random evolutionary steps.


      Now if you read Aquinas with a preconceived materialist notion, you might have missed the significance of what he expresses frequently in his work, which bears upon the observable randomness in nature.


      I wrote how an infinite, unbiased random number must contain all possible finite subsequences with the correct frequency (or it would be biased) and now, translating it as an understood ASCII type of expression, we can jump straight into Aquinas for a typically relevant comment:


      So, in Prima Pars, Question 14, Article 15 of the Summa Theologiae, he writes:


      "AD SECUNDUM dicendum quod compositio enuntiabilis significat aliquod esse rei, et sic Deus per suum esse essentia, est similitudo omnium eorum quae per enunttiabilia significuntur".


      In English translation this is:


      "Enunciatory composition signifies some existence of a thing; and thus God by his existence, which is His essence, is the similitude of all those things which are signified by enunciation."


      All of the finite random subsequences as ASCII are all enunciable by a large enough finite mind, and they are all subsequences of the infinite number, so the idea - that an infinite random number contains God's knowledge of everything that His finite creatures can express in words - is completely concordant with Aquinas's idea of our composite (this is the only way we can understand them) images of God.


      You should read it again, because sense can definitely be made of it, even though it doesn't necessarily fit what you have chosen to believe. It is amazing stuff.


      The Hound of Heaven is After you.


      https://www.bartleby.com/236/239.html


      Tom Cohoe

      Delete
    94. Most, if not all, of the ie's in that post should be eg's.

      Tom Cohoe

      Delete
    95. @ Don Jindra,

      Have you considered how, if Love is the fundamental principle of the universe, an infinite, unbiased, random sequence of binary digits, something apparently highly complex, could be an image of the most simple of all creative principles - uncreated, self subsisting Love, and how this image is so concordant with the various images of God put forth by Aquinas?

      Delete
  12. I dearly wish Ed would take on Schopenhauer's "World as Will and Idea", the First 2 Books, in which Shopenhauer discusses Matter and Materialism (along with Cartesian dualism, German Idealism, Spinozism etc).

    It is a master work, unifying Kant and Plato. Schopenhauer deals very little with Aquinas, and so I would love to hear from Ed on Schopenhauer's compatibilities and incompatibilities with Thomism.

    ReplyDelete
    Replies
    1. This or a post on Schopenhauer take on the psr or even a post on the old atheism series would be awesome.

      Delete
  13. Cartesian mechanical thinking is very useful for physics, chemistry and engineering. As an approach to investigating nature is has proven very fruitful. It doesn't become problematic until it is expanded into a metaphysics.

    But mechanical thinking is just one of several forms of thinking that is useful within its domain and problematic when expanded to other domains. Mathematical reasoning based on pure abstractions is useful, but Plato and many modern physicist take it too far, trying to give causal powers to abstractions. It then becomes a metaphysics and it fails.

    Another example is human action. If you want to know why things are the way they are, the question "who did that" is extremely useful in a wide variety of cases. Things happen because someone made them happen. But the value of these sorts of explanations is limited, and if you start populating the world with spirits to explain trees and springs, you have made it into a metaphysics. In modern life, you often see that sort of reasoning in conspiracy theories.

    And also, there is biological thinking, which is very useful for understanding growth and instinctive behaviors and the function of organs, but when you use it to try to explain the behavior of inanimate objects (such as each element seeking its own level), you have turned it into a metaphysics.

    ReplyDelete
  14. This is an insightful interview with theistic philosopher Trenton Merricks that touches on some of issues mentioned here, and some that are not.

    https://www.3-16am.co.uk/articles/propositions-truth-persons-and-vagueness?c=end-times-series

    ReplyDelete
  15. There's no hope for Western culture unless the materialist/physicalist tumour is extirpated.

    Thanks for doing your part Dr. Feser!

    ReplyDelete
    Replies
    1. No hope to rid itself of people who pride themselves of being 'free thinkers' while they acknowledge that they are literally slaves to the laws of physics up to the last atom of their bodies.

      But materialism/ physicalism is a weird cult that preys on weak-minded and gullible individuals.
      It's just a matter of time (pun intended) for it to disappear.

      It's killing its host.

      Delete
  16. Hi Ed,

    So let me get this straight. Qualities are not entirely mind-dependent. The fragrance of an apple is one of its qualities. Can you kindly explain what the mind-independent aspect of fragrance consists in?

    ReplyDelete
    Replies
    1. Don't forget the texture of an apple as well.

      Delete
    2. Vincent,
      When we took chemistry lessons, we had to observe the color and odor of gases. If a student put down that gas Z had an odor like that of pears when the actual odor was more like that of rotten eggs, the student's answer was marked wrong. In everyday work, chemists use the commonsense view that colors and fragrances inhere in the actual substances.

      Delete
    3. I think Vincent's point is that it seems nonsensical to suggest the specific quale of fragrance literally exists in a gas Z removed from our experiencing said quale. No one is denying that we roughly share the same qualia and that there are correlations between these and substances we study in the sciences (that wouldn't disturb a materialist, either).

      When we smell gas Z, we experience a certain fragrance, yeah. I can make sense of that.

      I cannot quite make sense of the suggestion that the qualitative fragrance exists in gas Z apart from my (or anyone else's) experiencing it, though.

      Delete
  17. Vincent,
    Let me put it another way.
    The mind independent aspect of fragrance is such that Ed and I and many other people could tell from fragrance alone whether the object we were smelling was whisky or brandy, and we would have a very good shot (if you don't mind the pun) at telling whether the object was Islay Scotch or Bourbon.

    ReplyDelete
    Replies
    1. Hi Tim,

      So you're saying that the mind-independent aspect of fragrance X is its similarity to fragrance Y, while the min-independent aspect of fragrance Y is its similarity to fragrance X? Sounds circular to me.

      Another problem with your proposal is that people don't smell things the same way, as the following short article from Science Daily about a new study of olfaction ("Do you smell what I smell?", 30 April 2019) reveals.

      "Humans have about 400 different types of specialized sensor proteins, known as olfactory receptors, in their noses. One odor molecule can activate several different olfactory receptors, while any given receptor can be activated by several different odor molecules. In a process that remains to be decrypted, the olfactory system somehow interprets these receptor activation patterns to recognize the presence, quality (does it smell like cherry or smoke?) and intensity of millions, maybe even trillions, of different smells...

      "Small differences in olfactory receptor genes, which are extremely common in humans, can affect the way each receptor functions. These genetic differences mean that when two people smell the same molecule, one person may detect a floral odor while another smells nothing at all."

      It was found that "a change in a single receptor was often sufficient to affect a person's odor perception." It turns out that some people may have a broken receptor, or one that functions poorly.

      So the similarities (or differences) in our fragrance perceptions really boil down to us having (or not having) the same olfactory receptors in their noses. But that's no longer a feature of the fragrance; it's a feature of the human nose.

      You might reply that receptors are activated by odor molecules, and that the min-independent feature of the fragrance consists in its being made up of certain molecules. But that's no longer a qualitative property. It's a chemical property. Also, there isn't a one-to-one mapping between odor molecules and olfactory receptors, as the article reveals.

      I'm having a very hard time seeing how the actual smell of whisky is something in the whisky. Instead, it sounds to me more like the human brain's interpretation of its molecular profile. Cheers.

      Delete
    2. That just describes a correlation which has sustained an inference for your idea that "whisky has such and such smell".

      Delete
    3. Hey Vincent, just a couple points. I’m not able to put myself one side of the fence or another on this issue, but it seems that subjectivity of experience is not proven by differing experiences of the same thing from different people given that there may be malfunctions of the human system. It may be the case that there are many elements of sense qualia being picked up at once, but someone may not pick them all up due to a lack of function. That does not mean that the quality does is not a property of the object. Further, even a weakened receptor can dull one smell and allow another to be the focus.

      I find that often unlikely things can smell similar. For example, everyone who I point this out to agrees with me, that skunk spray smells like burnt popcorn. Yet this is a matter of focusing on one element and ignoring others, and we would not say that a man is only subjectively good because I can focus on good qualities whilst ignoring his flaws.

      I think those are just two points to consider.

      Delete
  18. What's the matter with materialism? Well, the problem is that it lacks substance ;)

    Materialism/physicalism is just a fluffy, incoherent and corrupt philosophy.

    But "there's nothing new under the sun" and humans have been worshipping any kind of weird idol since their inception. For westerners now the idol is this misterious all-powerful all-creative all-encompassing "matter" of which "scientists" are their prophets. Put on a white labcoat and the idol shall communicate you all its secrets! Because "matter" loves to trade secrets with some random semi-evolved monkeys that are mere "accidents" in a "purposeless" Universe!


    ReplyDelete
  19. The smell of the whisky is there whether there is any human or any animal with the sense of smell in the room to smell it or not. That differences in olefactory genes affects some peoples experiences is irrelevant to what most people experience. People pay winemakers and perfume makers large amounts of money (in part) for a product with certain attributes. I dislike Veuve Cliquot but if it were on sale for $10 I would buy it and give it to a friend who liked the objective attribute that the wine has.

    ReplyDelete
    Replies
    1. Again, that there are objective correlations between our experience of a smell and the object itself, almost no one denies. Materialists, Cartesian dualists, property dualists and the like.

      But how to make sense of the idea of a *qualitative fragrance* of whisky being in whisky regardless of what anyone experiences is beyond me. You're free to just assert that there is a formal accident of "the qualitative smell of whisky" in a liquid, but you should understand others would find that assertion to be quite eccentric. It's like talking of a qualitative experience without an experiencer.

      Delete
    2. Yes there are objective properties that cause a particular experience of smell, but the experience of the small doesn’t exist without consciousness. There is nothing you could write, say or draw about the chemicals is whisky that would allow someone without a sense of smell to know what the experience of smelling whiskey is like. The smell itself is an experience, a partaking in the essence of the whiskey that cannot be captured by any instruments, only correlated afterwards.

      Delete
  20. If you claim that the whisky is colorless, odorless, tasteless, soundless, and lacking any tactile properties such as viscosity, you have fallen into the very trap that Democritus recognized but never solved. You rob the object (whisky in this case) of the very attributes that enabled you to learn whatever you learned about it. The senses proclaim: "Wretched mind, from us you are taking the evidence by which you overthrow us? Your victory is your own fall?"

    ReplyDelete
  21. Hi Tim,

    I don't claim that whisky is "colorless, odorless, tasteless, soundless, and lacking any tactile properties such as viscosity." Rather, what I claim is that color, odor, taste, sound and tactile properties don't apply to whisky as such. These phenomenal properties apply to human beings who are sensing whisky. (Alpha Centaurians presumably possess a completely different set of phenomenal properties.) But there is an objective reason why people who see, smell, taste, touch and hear the sound of whisky being poured experience these properties. The properties are grounded in the molecular structure of whisky itself. The Wikipedia article on viscosity explains that "the viscosity of a system depends in detail on how the molecules constituting the system interact." The molecular properties of whisky are objective. Cheers.

    ReplyDelete